Download as pdf or txt
Download as pdf or txt
You are on page 1of 165

1) 25 year teacher have fear attack and worry before enter the class ( I forgot all the

scenario) what is the initial treatment:


a. Selective serotonin reuptake inhibitor
b. Tricyclic depressant
c. Beta blocker
d. -----
e. ------
* Social phobia, known as: Social anxiety disorder
* common disorder characterized by excessive fears & embarrassment , leading to significant
distress or impairment in functioning
*Pathogenesis: experimental depletion of serotonin increase autonomic responses to fear
* In social situations: persons experience physical manifestations of anxiety, include blushing,
sweating, trembling, and palpitations, which sometimes can take the form of a full panic attack
* Treatment: Several classes of drugs are used, including selective serotonin reuptake inhibitors
(SSRIs), serotonin norepinephrine reuptake inhibitors (SNRIs), monoamine oxidase inhibitors
(MAOIs), beta-blockers, and benzodiazepines. We suggest SSRIs or SNRIs be used as first-line
medications
* SSRIs, SNRIs, and MAOIs all require up to 12 weeks to achieve full effect. Beta-blockers and
benzodiazepines achieve their effects within 30 to 60 minutes
* Selective serotonin reuptake inhibitors: paroxetine – sertraline – fluvoxamine – fluoxetine -
citalopram
-This is a case of social phobia and the initial treatment is answer a (SSRIs).
…………………………………………………………………………………………………………………………………………………

2) The best initial TTT for depression is:


a. SSRIs
b. Tricyclic depressant
c. MAO inhibitors
d. Beta blocker
e. -----------
* The major classes of drugs used to treat depression are considered "first generation"
(monoamine oxidase inhibitors [MAOIs], and tricyclic antidepressants [TCAs]) and "second
generation" (selective serotonin reuptake inhibitors [SSRIs], inhibitors of the reuptake of both
serotonin and norepinephrine [SNRIs], and a few drugs with unique modes of action)
* Most systematic reviews have concluded that clinical and quality of life outcomes, as well as
overall treatment costs, provide no clear guidance on the choice among specific antidepressants
* SSRIs are often first choice in primary care are fewer side effects and less danger with
overdose
-The correct answer is a (SSRIs).
…………………………………………………………………………………………………………………………………………………

3) Using the following classification :


Risk factor Case (disease) Non case total
Present A B A+b
Absent C D C+d
Total A+C B+D
Relative risk of those with risk factor to those without risk factor is:
a. A/A+B , C/C+D
b. A/A+B
c. C/C+D
-The correct answer is a.

Positive predictive value (PPV) : a / (a+b)


Negative predictive value (NPV) : d / (C+d)
Sensitivity : proportion of people with disease who have a positive test: a / (a+c)
Specificity : proportion of people without disease who have a negative test : d /(b+d)

Relative risk (RR) : incidence in expose ÷ incidence in unexposed: ( a / (a+b) ) / (c /


(c+d) )
Attributable risk : incidence in expose ‫ ـــ‬incidence in unexposed: ( a / (a+b) ) ‫( ــ‬c /(c+d) )
Odd ratio (OR) : Odds that a disease person is exposed ÷ Odds that a nondisease person
is exposed : ad / bc

4) 50 year old Man presented to ER with sudden headache, blurred of vision, and eye
pain. The diagnosis is:
a. Acute glaucoma
b. Acute conjunctivitis
c. Corneal ulcer
d. -----
* two main types of glaucoma: open-angle glaucoma and closed-angle glaucoma.
* Open-angle glaucoma ( 90% of glaucoma cases) : is painless and does not have acute attacks.
Just gradually progressive visual field loss
* Closed-angle glaucoma ( 10%): patients may experience some or all of the following
symptoms: • Decreased vision • Halos around lights • Headache • Severe eye pain • Nausea and
vomiting, with these Signs • Conjunctival redness • Corneal edema or cloudiness • A shallow
anterior chamber • A mid-dilated pupil (4 to 6 mm) that reacts poorly to light

-The correct answer is a.

…………………………………………………………………………………………………………………………………………………
5) Which heart condition is tolerable during pregnancy:
a. Eisenmenger syndrome
b. Aortic stenosis
c. Severe mitral regurge
d. Dilated cardiomyopathy with EF 20%
e. Mitral stenosis and the mitral area is 1 cm (or mm).
* Mitral stenosis is the most common valvular lesion of rheumatic origin found in women of
child-bearing age, followed by mitral regurgitation; aortic stenosis and regurgitation account for
the remainder of cases
* Regardless of etiology, pregnancy in the setting of Eisenmenger syndrome is associated with
significant morbidity and mortality
* The following settings were considered to be associated with high maternal and/or fetal risk:
- Severe aortic stenosis with or without symptoms
- Symptomatic mitral stenosis (NYHA class II to IV)
- Aortic or mitral regurgitation with NYHA class III to IV symptoms
- Aortic and/or mitral valve disease with severe left ventricular dysfunction (defined as an LVEF
less than 40 percent) or severe pulmonary hypertension (defined as pulmonary artery pressure
>75 percent of systemic pressure)
* The following settings were considered to be associated with low maternal and/or fetal risk:
- Asymptomatic aortic stenosis with an LVEF >50 percent
- mitral regurgitation with no or mild symptoms (NYHA class I to II)
-Mitral valve prolapse with either no mitral regurgitation or mild to moderate mitral regurgitation
associated with an LVEF >50 percent
- Mild mitral stenosis (defined as a mitral valve area >1.5 cm2 and a mean gradient less the 5
mmHg) without severe pulmonary hypertension (defined as pulmonary artery pressure >75
percent of systemic pressure)
- Mild to moderate pulmonary valve stenosis
* Pregnancy should be discouraged if there is a significant reduction in ventricular function
(ejection fraction <40 percent).
…………………………………………………………………………………………………………………………………………………
6) Diffuse abdominal pain “in wave like” and vomiting. The diagnosis is:
a. Pancreatitis
b. Appendicitis
c. Bowel obstruction
d. Cholelithiasis
 symptoms of the intestinal obstruction: Abdominal pain, distension, constipation (ealy in
distal obstruction), Vomiting (early in proximal)
The correct answer is c. (colicky pain: is the pain that comes in waves “wave-like” and it is
associated with bowel obstruction, cholelithiasis, nephrolithiasis)
…………………………………………………………………………………………………………………………………………………
7) Which type of contraceptive is contraindicative in lactation:
a. OCPs
b. Mini pills (progesterone-only birth control pills)
c. IUD
d. Condom
e. Depo-Provera (progesterone-only birth control injection)

-The correct answer is a. because combined OCPs decrease the quantity of milk.
Progesterone –only contraceptive are not believed to decrease the quantity of
milk.

8) A long scenario about patient with polydipsia ad polyuria. I don’t remember the
scenario but they mention osmolality in urine and serum, measurement of Na.
I encourage you to read about central VS nephrogenic diabetes insipidus.

- Q IS NOT COMPLETE .
- Polyuria: think of : 1) glucose-induced osmotic diuresis in uncontrolled diabetes mellitus
2) primary polydipsia 3) central DI; and 4) nephrogenic DI
- Primary polydipsia: increase in water intake with psychiatric illnesses
- low plasma sodium concentration (less than 137 meq/L) with a low urine osmolality (eg,
less than one-half the plasma osmolality) is usually indicative of water overload due to
primary polydipsia.
- A high-normal plasma sodium concentration (greater than 142 meq/L, due to water loss)
points toward DI, particularly if the urine osmolality is less than the plasma osmolality
- (Diabetes Insipidus) : failure to concentration of urine as a result of central or
nephrogenic ADH. Present with polyurea, polydepsia and hyponatremia.
Diagnosis central or nephrogenic by administration of Desmopressin acetate (DDAVP) ,
synthetic analog of ADH, can be used to distinguish central from nephrogenic DI.
1- Central DI : DDAVP challenge will decrease urine output and increase urine
osmalirity
2- Nephrogenic DI : DDAVP change will not significant decrease urine output
…………………………………………………………………………………………………………………………………………………
9) A 5 year old child came with earache on examination there is fluid in middle ear and
adenoid hypertrophy. Beside adenoidectomy on management, which also you should
do:
a. Myringotomy
b. Grommet tube insertion
c. Mastidectomy
d. Tonsillectomy
e. -----
- The diagnosis of AOM requires evidence of an acute history (Symptoms of AOM
include otalgia or ear pain, hearing loss, and vertigo ) , signs of middle ear inflammation
(distinct erythema of the tympanic membrane AND the presence of middle ear effusion
eg, tympanic membrane bulging, decreased or absent tympanic membrane mobility,
presence of an air-fluid level, or otorrhea)
- Treatment: 1) analgesics: ibuprofen or paracetamol
2) antibiotic: observation without use of antibacterial therapy is an option for selected
children with uncomplicated AOM , if complicated: amoxicillin remains the drug of
choice
3) recurrent and severe AOM or persistent OME: may require myringotomy and
placement of ventilating tubes (tympanostomy tubes : Grommet tube insertion )
* adenoidectomy should not be performed as the initial procedure in children with
persistent OME
- Myringotomy : Incision of the tympanic membrane usually for relief of pressure and
drainage of fluid from the middle ear : (is used for bulging acute otitis media)
-Grommet tube insertion (is used for recurrent acute otitis media : Children who have
OME of ≥4 months' duration with persistent hearing loss (≥21 dB) or other signs or
symptoms)
…………………………………………………………………………………………………………………………………………………

10) How the randomized control study become strong or of good validity: ( sorry I forgot
the answers)

Q IS NOT COMPLETE

-1) By doing blinding :( "If blinding done, who was blinded :" for example, participants,
care providers, those assessing outcomes" and how), RCTs without blinding are referred
to as "unblinded","open"

2) Allocation concealment (defined as "the procedure for protecting the randomisation


process e.g. by sequentially- numbered, opaque, sealed envelopes (SNOSE); sequentially-
numbered containers; pharmacy controlled randomization; and central randomization, the
aim is to reduce Selection bias

The best way to increase the power of the study is to increase the sample size.
…………………………………………………………………………………………………………………………………………………

11) Woman with postpartum depression, beside the medical TTT, whch should be include
in therapy:
a. Family therapy or support
b. -------

- Postpartum blues and depression:( depression that begins within the first month after delivery)
- treatment opitions:
Psychosocial therapy : (Interpersonal psychotherapy, Cognitive-behavioral therapy, Group
therapy, Family and marital therapy) - Light therapy - Pharmacologic therapy (Antidepressants )
- Electroconvulsive therapy - Hormonal therapy
12) Classic Scenario of stroke on diabetic and hypertensive patient. What is the
pathopysiology of stroke:
a. Atherosclerosis
b. Anyresm
c. -----
-The correct answer is a.
…………………………………………………………………………………………………………………………………………………

13) Middle aged patient with an acyanotic congenital heart disease the X-ray show
ventrical enlargement and pulmonary hypertension:
a. VSD
b. ASD
c. Trancus arteriosus
d. Pulmonary stenosis
- Atrial septal defects (ASDs) are the most common congenital lesion in adults after bicuspid
aortic valves. Although ASDs are often asymptomatic until adulthood, potential complications of
an undetected lesion include irreversible pulmonary hypertension, right ventricular failure.
- Symptoms: Atrial arrhythmias, exercise intolerance, fatigue, dyspnea, and overt heart failure,
Pulmonary hypertension and Eisenmenger syndrome are complications
-The correct answer is b, bcause the pt. in middle age and acynotic.
…………………………………………………………………………………………………………………………………………………

14) Best food in travelling is:


a. Boiling water
b. Water
c. Ice
d. Partial cocked fish and meat
e. ---
-The correct answer is a.
…………………………………………………………………………………………………………………………………………………

15) Mechanism of vitamin C in wound healing :


a. Epithiliazation
b. Aerobic fibroblast synthesis
c. Collagen synthesis
d. Enhance vascularization
e. ----
Think of Scurvy : a disease resulting from a deficiency of vitamin C, symptoms include skin
changes with roughness, easy bruising and petechiae, gum disease, loosening of teeth, poor
wound healing, and emotional changes
-The correct answer is c.
…………………………………………………………………………………………………………………………………………………
16) Open globe injury . TTT is:
a. Contineuos antibiotic drops
b. Continuous water and NS drops
c. Continuous steroids drops
d. Sterile cover and the refered
e. -----
-The correct answer is d.
17) Adolescent female counseling on fast food. What you should give her:
a. Ca + folic acid
b. Vit C + folic acid
c. Zinc + folic acid
d. Zinc + Vit C
-The correct answer is a.
…………………………………………………………………………………………………………………………………………………
18) What is the TRUE about backache with osteoporosis:
a. Normal x ray vertebra exclude the diagnosis
b. Steroid is beneficial TTT
c. Vit D defiency is the cause
d. Another TTT I don’t remember

- Osteoporosis is defined as a reduction in bone density (or mass) or the presence


of fragility fracture.
- Pathogensis: The underlying mechanism in all cases of osteoporosis is an imbalance between
bone resorption and bone formation, due to lack of estrogen, deficiency of calcium and vitamin
D and other ways

- Treatment: Oral calcium, vitamin D, exercise, and smoking cessation. Bisphosphonates


(alendronate , risedronate,ibandronate, zoledronic acid) , Raloxifene, a selective estrogen receptor
modulator

N.B: Defective mineralization of the organic matrix of bone results in osteomalacia

-The correct answer is c


…………………………………………………………………………………………………………………………………………………
19) Adolescent female with eating disorder and osteoporosis
a. Weight gain
b. Vit D
c. Bisphosphonates
d. --------
Eating disorders:
- Anorexia nervosa — four diagnostic criteria : 1) Refusal to maintain weight within a
normal range 2) Fear of weight gain despite being underweight 3) Severe body image
disturbance 4 ) postmenarchal females, absence of the menstrual cycle, or amenorrhea
- Bulimia nervosa: binge eating and inappropriate compensatory behavior to prevent
weight gain
- Eating Disorder Not Otherwise Specified
- Complications of anorexia nervosa include myocardial atrophy, mitral valve prolapse,
pericardial effusion, bradycardia, functional hypothalamic amenorrhea, antenatal and
postpartum problems, osteoporosis, gastroparesis, and constipation
-
-The correct answer is a.
…………………………………………………………………………………………………………………………………………………
20) Fourth degree hemorrhoids, Management is:
a. Hemoridectomy
b. band ligation
c. sclerotherapy
d. fiber diet
-The correct answer is a.

- Classification of internal hemorrhoid :


First degree: hemorrhoid do not prolapsed
Second degree: hemorrhoid prolapsed upon defecation but spontaneously reduce
Third degree: hemorrhoid prolapsed upon defecation, but must be manually reduce
Fourth degree: hemorrhoids are prolapsed and cannot be manually reduce

-treatment:
First & Second: life style modification (fiber dite)
Third: life style modification with (band ligation, sclerotherapy or cryotherapy) , if
Failed go to surgery.
Fourth: surgery (hemorrhiodectomy)

21) nulligravida at 8 weak gestational age, follow up for genetic screening, she refused
the invasive procedure but she agree for once screening , what is the appropriate
action now:
a. do ultrasound
b. 1st screening
c. 2nd screening
d. 3rd screening
e. Amncentesis
Genetic testing can be performed before birth using cells derived from the placenta (from 10 to 14
weeks of gestation), amniotic fluid (from 15 weeks of gestation to term), fetal blood (from about
18 weeks of gestation to term), or fetal tissue (from about 20 weeks of gestation to term), and
sometimes from cell-free fetal nucleic acids in maternal blood (from about 9 weeks of gestation)
If we think of Down syndrome:
The first trimester combined test ( performed at 11 to 13 weeks of gestation). It involves
sonographic determination of nuchal translucency (NT) and gestational age (by crown-rump
length) combined with the serum markers pregnancy-associated plasma protein-A (PAPP-A) and
human chorionic gonadotropin (hCG)

-The answer is correct B


…………………………………………………………………………………………………………………………………………………
22) 70 year old male with chronic Hepatitis B virus antigen carrier. The screening of choice
is:
a. Alfaprotien + liver ultrasound
b. Alfaprotien + another tumor marker
c. Abdominal CT + abdominal ultrasound
d. ----
-The correct answer is a.
…………………………………………………………………………………………………………………………………………………
23) 35 year old smoker , on examination sown white patch on the tongue, management:
a. Antibiotics
b. No ttt
c. Close observation
d. ---------
-This is a case of leukoplakia and the management includes:ask the pt. to stop
smoking, do a biopsy for the lesion; if there is pre-cancerous changes or cancer
in the biopsy ; surgical excision should be done.
- Leukoplakia — Oral leukoplakia is a precancerous lesion that presents as white patches
or plaques of the oral mucosa, Leukoplakia itself is a benign reactive process. However,
between 1 and 20 percent of lesions will progress to carcinoma within 10 years, The
clinical significance and natural history of oral leukoplakia depends upon the presence
and degree of dysplasia, Any indurated areas should be biopsied to rule out
carcinomatous changes
…………………………………………………………………………………………………………………………………………………

24) Regarding GDM:


a. Screening for GDM at 24 t0 28 weeks
b. Diet control is always successful TTT
c. Screening at 8 weeks
d. ---------
When to screen — Universal screening has traditionally been performed at 24 to 28 weeks of
gestation
How to screen –
1-h 50-g oral glucose tolerance test: screening (no fasting is needed), normal vaule < 140
3-h 100 –g OGTT: definitive, after an overnight FBS is draw, if > 125, means DM, if less than that
do this test and measure glucose level at 1,2 and 3, normal value is <180, <155, < 140, GDM is
diagnosed if 2 or more values are abnormal, if one is abnormal  impaired glucose intolerance
-The correct answer is a.
…………………………………………………………………………………………………………………………………………………

25) After doing CPR on child and the showing asystole:


a. Atropine
b. Adrenaline
c. Lidocane
d. ------

Asystole — Children with asystole have cardiac standstill with no discernible electrical activity
(figure 7). The most common cause is respiratory failure progressing to critical hypoxemia,
bradycardia, and then cardiac standstill. Underlying conditions include pneumonia, submersion,
hypothermia, sepsis, and poisoning (eg, carbon monoxide poisoning, sedative-hypnotics) leading
to hypoxia and acidosis

Patients with asystole or pulseless electrical activity should receive cardiopulmonary


resuscitation and epinephrine

-The correct answer is b.


26) Scenario of trauma , on face examination there is shifted mouth angle, loss of
sensation of ant. Third of tongue, which CN is affected:
a. Facial nerve
b. Trigeminal nerve
c. ----
-The correct answer is b , facial nerve for taste ant. Third of tongue, but
trigeminal nerve for sensation .
…………………………………………………………………………………………………………………………………………………

27) On examination of newborne the skin show papules or (pastules) over erythema base:
a. transient neonatal pustular melanosis
b. erythema toxicum neonatorum
c. ----
-The correct answer is b.
…………………………………………………………………………………………………………………………………………………

28) The most common causes of precocious puberty:


a. Idiopathic
b. Functional ovary cysts
c. Ovary tumor
d. Brain tumor
e. Adenoma

-The correct answer is a.


…………………………………………………………………………………………………………………………………………………

29) Mother worry about radiation from microwave if exposed to her child. What you tell
her:
a. Not all radiation are dangerous and microwave one of them
b. Microwave is dangerous on children
c. Microwave is dangerous on adult
d. ----
- In international study : all microwave is dangerous on children ( very vague Q )

…………………………………………………………………………………………………………………………………………………
30) Earlier sign of puberty in male is:
a. Appearance of pubic hair
b. Increase testicular size
c. Increase penis size
d. Increase prostate size
e. ---
-The correct answer is b.
31) Question about nutritional marasmus on definition.(protein energy malnutrition).

Marasmus is a form of severe protein-energy malnutrition characterized by


energy deficiency. A child with marasmus looks emaciated. Body weight may be
reduced to less than 80% of the average weight that corresponds to the height.
Marasmus occurrence increases prior to age 1, whereas Kawasaki occurrence
increases after 18 months. It can be distinguished from Kawasaki in that
Kawasaki is protein wasting with the presence of edema. The prognosis is better
than it is for Kawasaki.

…………………………………………………………………………………………………………………………………………………

32) What is the most risk of antihypertensive drugs on elderly patient:


a. Hypotension
b. Hypokalemia
c. CNS side effect
d. –
-The correct answer is a. because orthostatic hypotension will increase the risk
of fall and fractures in elderly.
…………………………………………………………………………………………………………………………………………………

33) Asymptomatic woman with trichomonas:


a. Treat if symptomatic
b. Treat if she is pregnant
c. Treat her anyway
d. –

-The correct answer is c.


Single dose of metradinazole for symptomatic and asymptomatic pt. and treatment
partner (because it is sexual transmitted disease). But, not treatment of
asymptomatic trichomniasis in first trimester of pregnancy. (Contraindication in
pregnancy)
…………………………………………………………………………………………………………………………………………………

34) What is the risk of GDM on her life later:


a. DM type 1
b. DM type 2
c. Impaired fasting glucose
-The correct answer is b.

35) What is the major sign that can tell you that patient have polycythemia vera rather
than secondary polysythemia:
a. Hepatomegaly
b. Splenomegaly
c. Venous engorgement
d. Hypertension
e. ----
-The correct answer is b.
in polycythemia vera there is clonal proliferation of a pluripotent marrow stem cell.
Also, level of erythropietine may be useful distinguish PCV, level are low, from
other causes of polycythemia. Secondary polycythemia is associated with excessive
dirusis, severe gastroenteritis and burn.
…………………………………………………………………………………………………………………………………………………
36) What is the deficient vitamin in infantile beri beri :
a. B1(thiamine)
b. C
c. E
d. Niacine
-The correct answer is a.

* VITAMINES DEFICIENCY :
Vit. A : night blindness & dry skin
Vit, B1 (thiamine): Beriberi (polyneuritis, dilated cardiomayopathy, edema)
Vit. B2 (riboflavin): angular stomatitis, cheilosis. Corneal vascuarization.
Vit. B3 (nacine) : pellagra ( Diarrhea, Dermatitis, Dementia )
Vit. B5 (pantothenate): dermatitis, enteritis, alopecia, adrenal insufficiency
Vit. B6 (pyridoxine): convulsion, hyperirritability
Vit B12 (cobalamin): macrocytic megaloblastic anemia, neurologic symptoms
FOLIC ACID : macrocytic megaloblastic anemia without neurologic symptoms.
Vit C : scurvy ( swollen gums, brusing, anemia, poor wound healing)
Vit D : rickets in children , osteomalasia in adult
Vit. E : increase fragility of RBC
Vit. K : neonatal hemorrhage, increase PT & PTT, normal BT
………………………………………………………………………………………………………………………………………………
37) 17 year boy admit to involve in recurrent illegal drug injection , what the screening
test to do:
a. HIV
b. Hepatitis B
c. Hepatitis C
d. -----
-All of the above choices are correct.?

38) 20 year old male found to have hepatitis b surface antibodies :


a. Previous vaccination
b. Previous infection
c. Active infection
d. ----
-The correct answer is a.

HBsAg : indicate carrier state.


HBsAb : indicate provide immunity to HBV
HBcAg: associated with core of HBV
HBcAB: during widow period, HBcAb-IgM indicate recent disease

…………………………………………………………………………………………………………………………………………………

39) Most common cause of secondary hypertension in female adolescent is:


a. Cushing syndrome
b. Hyperthyroidism
c. Renal disease
d. Essential HTN
e. Polycystic ovary disease
-The correct answer is c.
Most common causes of secondary HTN in young female is renal artery stenosis
( caused by fibromuscular dysplasia )
40) clomiphene citrate:
a. induce ovulation
b. ------
-The correct answer is a.
…………………………………………………………………………………………………………………………………………………

41) Scenario about perimenstrual depression syndrome ????

Premenestral syndrome : define as a symptoms complex of physiological emotional


symptoms sever enough to interfere with everyday life and occur cyclical during luteal
phase of menses
Premenestral dysphoric disorder : is a sever form of pf premenestral syndrome
characterized by severe recurrent depressive and anxity symptoms with premenestral (
luteal phase) onest that remit a few days after the start of menses.

…………………………………………………………………………………………………………………………………………………

42) Child present with stiffing neck, fever, headach. You suspect meningitis what is your
initial TTT should be:
a. Tobramycin
b. Levoflaxicine
c. Peneciline(ampicillin)
d. Doxycycline
 I remember there is no vancomycine, ceftriaxone or dexamethasone
on the choices.

-The correct answer is c.

AGE Causative organism Treatment

< 1 MONTH GBS, E coli Ampicillin + cefotaxime or gentamicin

1-3 MONTHS S.pneumonia, H.infulenza Vancomycin + cefotaxime or cefriaxone

meningocci

3 MONTHS - ADULT Pneumoccoci, meningococci Vancomycin + cefotaxime or ceftiaxone

>60 YEAR\acoholism Pneumococi, meningococci Ampicillin + vancomycin + ceotaxime or


Gram –ve bacilli ceftiaxone

43) A patient have tender, redness nodule on lacriminal duct site. Before referred him to
ophthalmologist what you will do:
a. Topical steroid
b. Topical antibiotics
c. Oral antibiotics
d. Nothing
e. –
The correct answer is c.(this is a case of dacrocystitis).
…………………………………………………………………………………………………………………………………………………

44) About DM in KSA:


a. about < 10 %
b. Most of the pt of insulin dependant type
c. female more affected with type 2 DM
d. most of NIDDM are obese

-The correct answer is d.


…………………………………………………………………………………………………………………………………………………

45) Major aim of PHC in Saudi Arabia :


a. To provide comprehensive maternal & child health
b. ------------
-other aims are:community participation, immunization, referral, chronic
disease management, prescribing, health education, management of
communicable disease and environmental health.
…………………………………………………………………………………………………………………………………………………
46) 17 y.o,she missed her second dose of varecila vaccine,the first one about 1 y ago what
you'll do:
a. give her double dose vaccine
b. give her the second dose only
c. revaccinate from start
d. see if she has antibody and act accordingly

-The correct answer is b.


…………………………………………………………………………………………………………………………………………………
47) there is outbreak of difteria and tetanus in community , regaring to pregnant woman:
a. contraindication to give DT vaccine
b. if exposed , terminate pregnancy immediately
c. if exposed , terminate after 72 hour
d. give DT vaccine anyway

-The correct answer is d.


48)All of the following are live vaccine except:
a. MMR
b. Oral plio
c. Varicella
d. Hepatitis B vaccine
e. BCG

-The correct answer is d.


…………………………………………………………………………………………………………………………………………………

49) Pt with scoliosis, you need to refer him to the ortho when the degree is:
a. 5
b. 10
c. 15
d. 20

-The correct answer is d.


…………………………………………………………………………………………………………………………………………………

50) 87 year old who brought by his daughter, she said he is forgettable, doing mess
thing in room , do not maintain attension , neurological examination and the
investigation are normal
a. Alzheimer disease
b. Multi-Infarct Dementia
c. ---

-The correct answer is a.


…………………………………………………………………………………………………………………………………………………

51) Mechanical intestinal obstruction


a. Nasogastric tube decompression

…………………………………………………………………………………………………………………………………………………

52) Patient with cancer. You want to break bad news, which of the following is the
answer:
a. Inform his family
b. Find out how much the patient know about it
c. Let social service inform him
d. Don’t tell him

-The correct answer is b.

53) HIV patient has hemorrhagic lesion in the mouth and papules in the face. Skin
biopsy show spindle cells and vascular structures:
a. Kaposi sarcoma
b. ………………….

Kaposi sarcoma : malignant multifocal neoplasm characterized by vascular tumors of


skin and viscera , caused by Human herpesvirus 8 (HHV8) , most commom associated
with AIDS patient. Lesion usally papules . in biobsy show spindle shape of cell.

…………………………………………………………………………………………………………………………………………………

54) Patient with retrosternal chest pain , barium swallow show corkscrew appearance
a. Achalasia
b. Esophagitis
c. GERD
d. Diffuse esophageal spasm

-The correct answer is d.

- Alchalasia : show esophagus dilation with (bird beak) tapering of distal esophagus
-Diffuse esophagus spasm: show corkscrew appearance.
…………………………………………………………………………………………………………………………………………………

55) Most common cause of intra cerebral hemorrhage:


a. ruptured aneurysm
b. Hypertension
c. Trauma
d. –
-The correct answer is b.

56) Rt upper qudrent pain and tenderness , fever, high WBC , jaundice, normal hepatic
marker
a. Acute cholecyctitis
b. Pancreatitis
c. Acute hepatitis
d. –

-The correct answer is a.


…………………………………………………………………………………………………………………………………………………
57) 5 y.o child with h.o fever and swelling of the face ant to the both ears (parotid
gland enlargement) what is the most common complication:
a. Labrynthitis
b. meningitis
c. encephalitis
d. orchitis.

-The correct answer is b.


Orchitis: (this one is the most common complication in postpubertal males-
adolscents and adults-). This pt. is prepuberty .

…………………………………………………………………………………………………………………………………………………
58) classic symptoms of tension headache .

Tension headach : is the most common type of headach diagnosed in adult. Present
with tight, band-like pain . may be generalized or most intense in the frontal,
occipital, and neck region. Usually occurs at the end of day. Treatmet avoidance of
exacerbating factors, NSAID and acetaminophen.

…………………………………………………………………………………………………………………………………………………
59) prophylaxis of Asthma
a. oral steroid
b. inhaler steroids
c. inhaler bronchodilator B agonists

-The correct answer is b.


…………………………………………………………………………………………………………………………………………………
60) 30 year woman with dysmenorrhea, menorrhagea, infertility, and on examination
found immobile mass on uterosacral ligaments :
a. uterine fibroid
b. endometriosis
c. ---
-The correct answer is b
61) Classic symptoms of primary dysmenorrheal

Primary dysmenorrhea: menestral pain associated with ovulatory cycle in abcense


of pathological finding.
Secondary dysmenorrhea: Menestral pain wich organic causes exist, most common
cused by endometriosis, adenomyosis, tumor and fibroid.
…………………………………………………………………………………………………………………………………………………

62) Newly married woman complain of no pregnancy for 3 month with unprotective
sexual intercourse :
a. Try more ( infertility is defined as no pregnancy for one year)

…………………………………………………………………………………………………………………………………………………

63) Younger diabetic patient came with abdominal pain, vomiting and ketones
smelled from his mouth. What is frequent cause:
a. Insulin mismanagement
b. Diet mismanagement
c. ----
-The correct answer is a.. But if the infection is one of the answers , you should
choose it because the most common cause of DKA is infection.
…………………………………………………………………………………………………………………………………………………

64) Cause of syncope in aortic stenosis


a. Systemic hypotension

………………………………………………………………………………………………………………………………………………
65) On stroke management: except
a. IV fluid not include dextrose
b. Diazepam for convulsion
c. No indication of anticonvulsive drugs
d. ----

-The correct answer is c.


…………………………………………………………………………………………………………………………………………………
66) Patient use illegal drug abuse and the blood show RNA virus. Which hepatitis
a. A
b. B
c. C
d. D

-The correct answer is c.


67) Classic symptoms and signs of infectious mononucleosis (EBV) ?

Most common occur in young adult, usually cused by EBV , transmission through
exchange of body fluid include saliva. Present with triad ( fever, sorethroat,
lymphadenopathy) , also, may present tonsillar exudates, splenomegaly,
maculopapular rash and bilateral upper eyelid edema. Management is mostly
supportive and corticosteroids are indicated for airway compromise due to tonsillar
enlargement.

…………………………………………………………………………………………………………………………………………………

68) Treatment of EBV ( in scenario there patent with tonsiller exudates,


lymphadenopathy, splenomegaly) :
a. Oral acyclovir
b. Oral antibiotic
c. IM or IV acyclovir
d. Supportive TTT
e. Observation

-The correct answer is d.


…………………………………………………………………………………………………………………………………………………

69) 25 year old woman with weight loss, heat intolerance, irritable ….
a. Hyperthyroidism

…………………………………………………………………………………………………………………………………………………

70) Patient with coloured pastules around his mouth, organism show herpes simplex
type 1, what is the treatment:
a. Oral antiviral
b. Iv antiviral
c. Supportive
d. –

-The correct answer is a.


…………………………………………………………………………………………………………………………………………………

71) Acute otitis media , initial TTT:


a. Amoxicilline

72) Man with sudden onset of scrotal pain , also had Hx of vomiting, on examination
tender scrotom and there is tender 4 cm mass over right groin, what you will do:
a. Consult surgeon
b. Consult urologist
c. Do sonogram
d. Elective surgery

-The correct answer is b , if expected tortion of tests , don't wast time on


radiological study
…………………………………………………………………………………………………………………………………………………
73) Most common symptoms of renal cell carcinoma is
a. Hematuria
b. Abdominal mass
c. Flank pain
-The correct answer is a.
…………………………………………………………………………………………………………………………………………………
74) Link the ttt with organism:
a. Shegella  metronidazole
b. Salmonella  erythromycin
c. Combylobacter  amoxacilline
d. Gardia  I foregut ( but I remember it is antibiotic)

-All of the above answers are wrong.?


…………………………………………………………………………………………………………………………………………………
75) Medical student had RTA systolic pressure is 70 mmhg, what you will do next in
management:
a. IV fluid therapy
b. ECG
c. Abdominal U/S
d. ---
-The correct answer is a.
…………………………………………………………………………………………………………………………………………………
76) 20 year old male had been stabbed on midtriceps , ‫المهم لفاها وبعد أسبوع فتحوا الجرح‬
‫وطلع سائل أخضر‬.
On microscopic examination of this greenish fluid show gram positive cocci in
chains:
a. Streptococcal gangrene
b. Chlostrideal gangrene
c. Fournier’s gangrene
d. meningocemia
e. –
-The correct answer is a.
77) Patient around his nose there are pastules, papules and telangiectasia lesions. The
diagnosis is:
a. Rosacea

…………………………………………………………………………………………………………………………………………………

78) Child with radial head dislocation, what is the next in management:
a. Reduction
b. x ray
c. MRI
d. –

-The correct answer is a.


…………………………………………………………………………………………………………………………………………………

79) In cervical LNs there are well differentiated thyroid cells, during operation you find
no lesion on thyroid what will you do next
a. Total thyroidectomy
b. Total thyoidectomy + radical cervical LNs dissection
c. Total thyroidectomy + specific LNs dissection
d. Thyoid lobectomy with -----

-The correct answer is b.


…………………………………………………………………………………………………………………………………………………

80) Irritable bowel syndrome , ( they mention a specific mechanism” I do not


remember” that produce which symptom
a. Vomiting
b. Diarrhea
c. Constipation
d. Abdominal pain
???????????? Q IS NOT CLEAR
…………………………………………………………………………………………………………………………………………………
81) Free fluid accumulate in abdominal cavity cause:
a. Hypovolemic shock
b. Cardigenic shock
c. Sepsis
d. Emesis
e. ---

-The correct answer is c.


82) Woman with cyclic bilateral nodularity in her breast since 6 month, on
examination there is 3 cm tender mobile mass in her breast : what you will do
next
a. FNA with cytology
b. Mammogram
c. Biopsy
d. Follow up for next cycle
e. Observation

-The correct answer is d. ( I'm not sure )


…………………………………………………………………………………………………………………………………………………

83) Most common symptoms of soft tissue sarcoma :


a. Paralysis
b. Ongrowing mass
c. Pain
d. ---

-The correct answer is b.


Ongrowing mass: painless and slow-growing.
…………………………………………………………………………………………………………………………………………………

84) the most common symptom in placenta abruption is


a. Vaginal bleeding
b. Uterine tenderness
c. Uterine contractions
d. Fetal distress

The correct answer is a

Placental abruption is mainly a clinical diagnosis with all the above findings. the
most common symptom is dark red vaginal bleeding with pain during the third
trimester of pregnancy (80%) and abdominal or uterine tenderness (70%). Bleeding
may occur at various times in pregnancy Bleeding in the first trimester of pregnancy
is quite common and may be due to the following: miscarriage (pregnancy loss)
ectopic pregnancy (pregnancy in the fallopian tube) . Bleeding in late pregnancy
(after about 20 weeks) may be due to the following: placenta previa or placental
abruption.
85) Female presented with vaginal discharge, itching, and on microscope showed
mycoleous cells and spores. This medical condition is most likely to be associated
with:
a. TB
b. Diabetes
c. Rheumatoid Arthritis

The correct answer is b

Vaginal thrush is a common infection caused by a yeast called Candida albicans.


Vulvovaginal candidiasis is usually secondary to overgrowth of normal flora Candida
species in the vagina. Conditions that interrupt the balance of normal vaginal flora
include: antibiotic use, oral contraceptives, contraceptive devices, high estrogen levels,
and immunocompromised states such as diabetes mellitus and HIV. Women are prone
to vaginal thrush between puberty and the menopause because, under the influence of
the hormone estrogen, the cells lining the vagina produce a sugar and yeasts which
Candida albicans are attracted to. That is why thrush is rare before puberty.
…………………………………………………………………………………………………………………………………………………
86) Primigravida in her 8th week of gestation, presented to your clinic wanting to do
genetic screening, she declined invasive procedure . the best in this situation is

a. Amniocentesis
b. 1st trimester screening
c. 2nd trimester screening
d. Ultrasound

The correct answer is b


…………………………………………………………………………………………………………………………………………………

87) Trichomoniasis is classically have:


a. Clue cells
b. Greenish frothy discharge

The correct answer is b

Trichomoniasis is caused by the flagellated protozoan Trichomonas vaginalis; it's


the most common curable sexually transmitted disease in the world. It usually
presents with frothy yellow-green vaginal discharge, strong-unpleasant odor, pain
during urination and sexual intercourse.
…………………………………………………………………………………………………………………………………………………

87) Obsessive neurosis:


a. Treatment is east
b. Clomipramine doesn’t not work
c. Mostly associated with severe depression
d. Can be cured spontaneously
the correct aswer is c
88) Patient came to you complaining of hearing voices, later he started to complain of
thought get into his mind and can be taken out
a. SCZ
b. Mood
c. Mania
d. Agoraphobia

The correct answer a


…………………………………………………………………………………………………………………………………………………

89) Female had history of severe depression, many episodes, she got her remission for
three months with Paroxitine ( SSRIs) .. now she is pregnant .. your advise
a. Stop SSRi's because it cause fetal malformation
b. Stop SSRi's because it cause premature labor
c. Continue and monitor her depression
d. Stop SSRIs

The correct answer is c or a

In general, SSRI have the least side effects then other classes of antidepressants.
Some SE: Sleep disturbance, dizziness, sexual dysfunction, Nervousness, and
diarrhea
……………………………………………………………………………………………………………………………………………
90) Hallucinations and Paranoia:

a. SCZ
b. Mood
c. Mania
d. Phobia

The correct answer is a


…………………………………………………………………………………………………………………………………………………

91) Female presented with thirst and polyurea.. all medical history is negative and she
is not know to have medical issues.. .she gave history of being diagnosed as
Bipolar and on Lithium but her Cr and BUN is normal. What is the cause of her
presentation
a. Adverse affect of lithium
b. Nephrogenic DI
c. Central DI

The correct answer is a

Nephrogenic diabetes insipidus is characterized by a decrease in the ability to


concentrate urine due to a resistance to ADH action in the kidney. Nephrogenic
diabetes insipidus can be observed in chronic renal insufficiency, lithium toxicity,
hypercalcemia, hypokalemia.
92) Most common medical problems encountered in Primary care is:
a. Coryza
b. UTI
c. HTN

The correct answer is a


…………………………………………………………………………………………………………………………………………………

93) Regarding diabetes mellitus in pregnancy :


a. Prevelance of diabetes mellitus in pregnancy is 10%
b. Glucose screeing is best in 24-28 week
c. Diabetic and non diabetic have same perinatal mortality
d. Gestational diabeted can be diagnosed by abnormal FGS test

The correct answer is b


…………………………………………………………………………………………………………………………………………………

94) Left red eye, watery discharge, photo phobia, peri-auricular non-tender lymph
nodes .. Dx
a. Bacterial conjctvitis
b. Viral conjctvitis

The correct answer is b


…………………………………………………………………………………………………………………………………………………

95) One of the Anti-psychotics causes ECG changes , Leukopenia, drooling :

a. Respiredone
b. Colzapine
c. Amisulpride

The correct answer is b


…………………………………………………………………………………………………………………………………………………
96) One of the following decrease chance of colon cancer :

a. Zinc
b. Vit. E
c. Vit C
d. Folic acid

????????

Both C, E, Beta carotene all are Antioxidants, they are correct but C is more
likely. A big dose of vitamin C fights the big "C." some others: Fiber ,Vitamin C,
Calcium, Vitamin E, Selenium.
97) Best sentence to describe specifity of screening test,is the population of people
who :
a. Are negative of disease, and test is negative
b. Are positive of disease, and test is negative
c. Are positive comparing to total other people
d. Negative disease , positive test
e. Positive disease , negative test

The correct answer is a

Specificity measures the proportion of negatives which are correctly identified


(e.g. the percentage of healthy people who are correctly identified as not having
the condition).
Sensitivity (also recall rate) measures the proportion of actual positives which
are correctly identified (e.g. the percentage of sick people correctly identified as
having the condition).
…………………………………………………………………………………………………………………………………………………

98) Heavy smoker came to you asking about other cancer, not Lung cancer, that
smoking increase its risk:
a. Colon
b. Bladder
c. Liver

The correct answer is b

smoking-associated cancers are lung, head &neck (like esophagus and larynx),
bladder and kidney, pancreatic, cervical and stomach.
…………………………………………………………………………………………………………………………………………………

99) Mid clavicle fracture :


a. Surgery is always indicated if fracture is displaced
b. Figure-8-dressing has better outcomes than simple sling
c. Figure-8-dressing is strongly indicated in patient with un-union risk
d. Both figure-8 and simple sling has similar outcomes

the correct answer is d


- simple sling has been to give the same result as a figure-8 (more comfort and
fewer skin problem).
…………………………………………………………………………………………………………………………………………………

100) patient complains of "can't breathe air in one nostril "; on examination showed
edematoud mucosa structure, best to give initially :
a. Corticosteroids
b. Decongestants
c. Alfa-adrenergic blockers
The correct answer is a
101) Pediatric came to you in ER with wheezing, dyspnea, muscle contraction ( most
probably asthma), best to give initially is :
a. theophillin
b. Albuterol neubelizors
c. oral steroids

the correct answer is b

………………………………………………………………………………………………………………………………………………

102) Female presented with complain of neck pain and occipital headache , no other
symptoms , on X-ray has cervical spine osteophyes and narrow disks :
a. cervical sponylosis ?

??????
…………………………………………………………………………………………………………………………………………………

103) Patient complaining of pain at night when he elevated his arm, tingeling on lateral
arm side and lateral three fingers , Dx
a. brachial plexus neuropathy
b. shoulder impengment syndrom
c. brachial artery thrombophebitis
d. thoracic outlet problem

the correct answer is d

Branchial plexus neuropathy is characterized by acute onset of intense pain in the


shoulder or arm followed shortly by focal muscle weakness.

Impingement syndrome, swimmer's shoulder, and thrower's shoulder, is a clinical


syndrome which occurs when the tendons of the rotator cuff muscles become
irritated and inflamed. This can result in pain, weakness and loss of movement at
the shoulder. The pain is often worsened by shoulder overhead movement and may
occur at night, especially if the patient is lying on the affected shoulder.

Thoracic outlet syndrome: numbness and tingling in the fingers; pain in the neck,
shoulder, and arm; weakness of the arm and dropping things from the hand;
worsening of the symptoms when elevating the arm to do such things as comb or
blow dry one's hair or drive a car; and coldness and color changes in the hand. The
symptoms are often worse at night or when using the arm for work or other
activities.
104) Young adult presented with pain on lateral elbow, tingeling of lateral arm, he
plays Squash:
a. carbel tunnel
b. tennis elbow

the correct answer is b


-Lateral epicondylitis( inflammation of common extensor tendon ) also known
as (tennis elbow, shooter's elbow and archer's elbow is a condition where the
outer part of the elbow becomes sore and tender. It is commonly associated
with playing tennis and racquet sports.

-Medial epicondylitis (inflammation of common flexor elbow) also know (golfer


elbow)
…………………………………………………………………………………………………………………………………………………

105) male came to you complaining of sudden progressive decreasing in vision of left
eye over last two/three days, also pain on the same eye, on fundoscopy optic disk
swelling was sees , Dx :
a. central retinal artery occlusion
b. central retinal vein occlusion
c. optic neuritis
d. macular degeneration

they said the correct answer is c

 Ask about symptoms of temporal arteritis in the older population. Patients complain of
sudden, painless, nonprogressive vision loss in one eye. History of headaches, jaw
claudication, scalp tenderness, proximal muscle and joint aches, anorexia, weight loss,
or fever may be elicited.
 Some patients may reveal a history of amaurosis fugax involving transient loss of vision
lasting seconds to minutes but which may last up to 2 hours. The vision usually returns
to baseline after an episode of amaurosis fugax.
 Central retinal Artery occlusion: presenting complaint is an acute sudden painless
unilateral loss of vision. Shows a classic cherry red spot
 Central retinal vein occlusion: usually sudden painless variable visual loss; the fundus
may show retinal hemorrhages, dilated tortuous retinal veins, cotton-wool spots,
macular edema, and optic disc edema.
 Optic neuritis: Major symptoms are sudden loss of vision (partial or complete), or
sudden blurred or "foggy or washed out" vision, and pain on movement of the affected
eye. Many patients with optic neuritis may lose some of their color vision in the affected
eye (especially red). The optic disk becomes swollen
 Macular degeneration is a leading cause of permanent painless irreversible vision loss in
the elderly.

Retinal Detachment; Symptoms are decreased peripheral or central vision, often described as a
curtain or dark cloud coming across the field of vision. Associated symptoms can include
painless vision disturbances, including flashing lights and excessive floaters.
106) unilateral headache, exaggerated by excersice and light , Dx :
a. migraine
b. cluster headach
c. stress headache

the correct answer is a


…………………………………………………………………………………………………………………………………………………

107) 70 years old with progressive demntia , no personality changes , neurological


examination was normal but there is visuodeficit , on brain CT shower cortex
atrophy and ventricular dilatations :
a. multi micro infract dementia
b. alzehimer demenita
c. parkinsonism dementia

the correct answer is b

 alzehimer dementia :
most common cause of dementia. age and family history are risk factors for AD.
Etiology unknown but toxic b-amyloid deposit in brain. Present with amnesia for
newly acquired information is usually the first presentation, followed by language
deficit , acaluia, depression, agitation and finally apraxia(inability to perform skilled
movement). Diagnosis by exclusion that can be definitive diagnosis only on
autopsy: suggested by clinical feature and by progressive cognitive course without
substantial motor impairment. MRI & CT may show atrophy , venticule
enlargement and can rule out other causes. On brain microscopy amyloid plaques
and neurofibrially tangle. Death usually occurring secondary to aspirati on
pneumonia . treatment by supportive therapy for Pt. and family , and
cholinesterase inhibitor .

 multi mico infarct dementia ( vascular demensia )


dementia associated with history of strok. Criteria for vascular dementia include
presence of dementia and 2 or more of the following:
1- focal neurological signs
2- symptoms onset that was abrupt , stepwise, or related to strok
3- brain imaging showing evidence of fold infarction or extensive deep white matter
changes secondary to chronic ischemia.

…………………………………………………………………………………………………………………………………………………

108) 70 years old with progressive dementia , on brain microscopy amyloid plaques
and neurofibrillary tangles are clearly visible also Plaques are seen : Dx
a. lewy dementia
b. Parkisonism
c. Alzehimer

The correct answer c


109) after bite, pediatric patient presented with abdominal pain and vomiting , stool
occult blood , rash over buttock and lower limbs , edema of hands and soles ,
urine function was normal but microscopic hematurea was seen:
a. Lyme
b. Henoch-Schonlein Purpura

The correct answer is b


Henoch-Schönlein purpura (HSP) is a small-vessel vasculitis characterized by
purpura, arthritis, abdominal pain, and hematuria.
…………………………………………………………………………………………………………………………………………………

110) for the above disorder (Q 109 ), which one is considered pathological
a. gross hematurea
b. microscopic hematurea
c. rashes
d. ..

The correct answer is a (I'm not sure )

…………………………………………………………………………………………………………………………………………………

111) Young adult presented with painless penile ulcer rolled edges .. what next to do :
a. CBC
b. Darkfeild microscopy
c. Culturing

The correct answer is b

Syphilis also known as "great imitator" is a sexually transmitted disease caused


by the spirochete bacteria Treponema pallidum. Classically presents as single
painless non-itchy skin ulceration with sharp borders. T pallidum is too small to
be seen under the light microscope. So use darkfield microscopy when sores are
present. Blood tests can confirm the presence of antibodies. The antibodies
remain in your body for years, so the test can be used to determine a current or
past infection.
…………………………………………………………………………………………………………………………………………………

112) Diabetic female ger 24h-urine proteinn is 150mg ,,


a. start on ACEIs
b. refer to nephrologist
c. do nothing , this is normal range

the correct answer is a


113) which prevent or decrease incidence of getting post herpetic neuralgia
a. Amitriptylin
b. Acyclovir
c. Varicella vaccination

The correct answer is c

All are true, but if I have to choose the best it will be varicella vaccination,
because it completely prevents.

…………………………………………………………………………………………………………………………………………………

114) Adult Polycystic kidney mode of inheritance:


a. Autosomal dominant

…………………………………………………………………………………………………………………………………………………

115) Female came with complain of diahrrea in the last 6 months, she lost some
weight, she reported that mostly was bloody , when you preformed
sigmoidoscopy you found fragile mucosa with bleeding ,Dx
a. colon cancer
b. Chron's
c. Ulcerative colitis
d. Gastroenteritis
e. Hemrrohids

The correct answer is c

…………………………………………………………………………………………………………………………………………………

116) Anal fissure commonest site


a. Posteriorly

…………………………………………………………………………………………………………………………………………………

117) mother gave bitrh of baby with cleft lip and palate, she want to get pergnant
again what is the percentage of recurrence
a. 1%
b. 4%
c. 15%

The correct answer is b


118) 4 years old girl presented with her parents to er with sore throat and seroangious
vaginal discharge with no pain what is most propable cause,
a. Candida
b. Foreign body
c. Chlamedia
d. Gonococci
e. streptococcus
The correcr answer is e

 Pediatric vaginal discharge:


1- infectious vulvovaginitis: present with malodorous , yellow green, most common caused by
group A streptococcus. ( may be present with sexual abus "STDs" )
2-foreign body
3-candidal infection: may associated with diabetes … measure glucose.
4-Sarcoma botryoids (rhabdomyosarcoma ): malignant lesion appearance of " bunches of
graps" within vagina
………………………………………………………………………………………………………………………………………………

119) patient coplaining of pain along median nerve ditribution , And positive tinel sign
treatment include casting of both hand in what position
a. Dorsiflexion
b. plantar flexion
c. extention
d. Dduction

The correct answer is c


…………………………………………………………………………………………………………………………………………………

120) dermatomyosistis what is true


a. distal muscle weakness
b. Underlying malignancy
c. Generalized ??? Skin rash

The correct answer is c

 Polymositis & dermatomyositis :


Polymyositis : progressive , systemic tissue disease characterized by immune-mediate
striated muscle inflammation, present with symmetric progressive proximal muscle
weaknes and pain .

DERMATOMYOSITIS : present polymyositis plus cutaneous involvement, heiotrop rash


(violaceous periorbital rash) , shawl sign ( rash involving the shoulder, upper chest and
back ) , Gottron"s papule ( popular rash with scale ).

-both : increase serum CK and anti-Jo antibodies . muscle biopsy uscle fiber and
inflammation.
121) pt with hypertrophic subaortic stenosis referred from dentist before doing dental
procedure what is true
a. 50 % risk of endocarditis
b. 12 % risk of endo carditis
c. No need for prophylaxis
d. post procedure antibiotic is enough

the correct answer is c


…………………………………………………………………………………………………………………………………………………

122) pt want to quit smoking you till him that symptoms of nicotine withdrawal
appear after
a. 1-2 days
b. 2-4 days
c. 5-7 days
d. 8- 10 days

The correct answer is a

…………………………………………………………………………………………………………………………………………………

123) pt taking bupropion to quit smoking what is SE


a. Arrythmia
b. Seizure
c. xerostomia
d. Headache

The correct answer is b

…………………………………………………………………………………………………………………………………………………

124) 14 years old girl complaining of painless vaginal bleeding for 2-4 days every
3Weeks to 2 months ranging from spotting to 2 packs per day; she had 2ry sexual
ccc 1 year ago and had her menstruation since 6 months on clinical examination
she is normal sexual ccc, normal pelvic exam appropriate action
a. OCP can be used
b. You should ask for FSH and prolactin level
??????????????
125) pt want to do dental procedure , he was dx to have mitral valve prolapse clinically
by cardiologist, he had never done echo before what is appropriate action
a. Do echo
b. No need for prophelaxis
c. give ampicillin
d. Give amoxicillin calvulinic

I think I will do Echo, but this are some of the information I found so I cant decide.

This is from MYO clinic

Doctors used to recommend that some people with mitral valve prolapse take
antibiotics before certain dental or medical procedures to prevent endocarditis, but not
anymore. According to the American Heart Association, antibiotics are no longer
necessary in most cases for someone with mitral valve regurgitation or mitral valve
prolapse.

Still, if you've been told to take antibiotics before any procedures in the past, check with
your doctor to see how these new recommendations apply to you.

This is from MedicinNet

The vast majority of patients with mitral valve prolapse have an excellent prognosis and
need no treatment. For these individuals, routine examinations including
echocardiograms every few years may suffice. Mitral regurgitation in patients with
mitral valve prolapse can lead to heart failure, heart enlargement, and abnormal
rhythms. Therefore, mitral valve prolapse patients with mitral regurgitation are often
evaluated annually. Since valve infection, endocarditis, is a rare, but potentially serious
complication of mitral valve prolapse, patients with mitral valve prolapse are usually
given antibiotics prior to any procedure which can introduce bacteria into the
bloodstream. These procedures include routine dental work, minor surgery, and
procedures that can traumatize body tissues such as colonoscopy, gynecologic, or
urologic examinations. Examples of antibiotics used include oral amoxicillin and
erythromycin as well as intramuscular or intravenous ampicillin, gentamycin, and
vancomycin.

…………………………………………………………………………………………………………………………………………………

126) ttt of cyclical mastalgia


a. OCP, analgisc, NSAID, Fat reduction, and magnisuem

Mastalagia : painful breast tissue that can be cyclic and usually associated with
hormonal change, often bilateral . management : stop current hormonal therapy
, reassurance, stop smoking, fat reduction, analgesic, NSAID, OCP.
127) 4years old child what can he do
a. Copy square and triangle
b. Speak in sentences
b. ..

The correct answer is a

…………………………………………………………………………………………………………………………………………………

128) baby can sit without support, walk by holding fourniture. Pincer grasp, pull to
stand how old is he
a. 8 months
b. 10 months
c. 12 month
d. 18 month

The correct answer is b

…………………………………………………………………………………………………………………………………………………

129) repeated question about Alzheimer disease

 alzehimer dementia :
most common cause of dementia. age and family history are risk factors for AD.
Etiology unknown but toxic b-amyloid deposit in brain. Present with amnesia for
newly acquired information is usually the first presentation, followed by language
deficit , acaluia, depression, agitation and finally apraxia(inability to perform skilled
movement). Diagnosis by exclusion that can be definitive diagnosis only on
autopsy: suggested by clinical feature and by progressive cognitive course without
substantial motor impairment. MRI & CT may show atrophy , venticule
enlargement and can rule out other causes. On brain microscopy amyloid plaques
and neurofibrially tangle. Death usually occurring secondary to aspiration
pneumonia . treatment by supportive therapy for Pt. and family , and
cholinesterase inhibitor .

…………………………………………………………………………………………………………………………………………………

130) repeated Q also about multiinfarct dementia


 multi mico infarct dementia ( vascular demensia )
dementia associated with history of strok. Criteria for vascular dementia include
presence of dementia and 2 or more of the following:

1- focal neurological signs


2- symptoms onset that was abrupt , stepwise, or related to strok
3- brain imaging showing evidence of fold infarction or extensive deep white matter
changes secondary to chronic ischemia.
131) 73 year patient complain of progressive loses of memory with decrees in
cognition function . C.T reveal enlarge ventricle and cortical atrophy , diagnosis is

a- Alzheimer
b-multi infarct dementia
c- multiple sclerosis
d-????????

The correct answer is a

 Alzheimer (Dx by exclusion. Its associated with progressive memory loss,


decreased cognition function , & enlarged ventricles with cortical atrophy)
 multi infarct dementia (NOT progressive & it has focal neurological abnormality)
 multiple sclerosis (recurrent relapsed & complete remission. Its associated with
demylenation of gray-matter)

…………………………………………………………………………………………………………………………………………………

132) 62 female with –ve pap smear you should advice to repeat pap smear every:
A- 6m
b- 12m
c- 18m
d- no repeat

The correct answer is d

Screening pap smear:


1- starting at age 21 years or no more than 3 years after becoming sexually active.
2- women > 30 years who have three consecutive normal test screening ( 1 / 3yeasr).
3- screening should be discontinue for women > 60-70 years who have had 3 or more
normal pap smear.

…………………………………………………………………………………………………………………………………………………

133) all following are criteria of chronic fatigue syndrome except


b-???????

chronic fatigue syndrome:

characterizes by profound mental and physical exhaustion. In associated with


multiple system and neurospsychiatric symptoms that last at least 6 mounth. Must
be new ( not life long ) , must not be relived by rest, and must result in greater than
50% reduction in previous activity. Presentation with 4 or more of the following :
poor memory / concentration, myalgia, arthalagia, sore throat, tender lymph node,
recent onset headach, unrefreshing sleep, excessive tiredness with exercise.
Treatment by : cognitive and excercise therapy .also, diet, physiotherapy, dietary
supplements, antidepressants
134) 12 year boy with sinsoneurl hear loose , C.T scan show mass (site in brain???) so
diagnosis is
a- aqustic neroma
b-meningioma
c- barotraumas
d????????

The correct answer is a (I'm not sure )

- aqustic neroma (benign tumor of cranial nerve VIII. Mostly occur between age of 30-60 years.
, Mostly unilateral except if it's associated with neurofibromatosis in which its bilateral)
-meningioma (mostly benign,  with age, more with female. Its occur in the cerebellopontin)
- barotraumas (mostly in the divers. The damage occur due to  pressure)

..……………………………………………………………………………………………….………………………………………………

135) 50 y with uncontrolled diabetes ,complain of black to brown nasal discharge. So


diagnoses is
a- mycomyosis
b- aspirglosis
c-foreign body
d????????

The correct answer is a

- mycomyosis (fungal infection caused by Mycorales, affect nasal sinus & lungs, .
…………………. characterized by black nasal discharge, Dx by biopsy).
………………………………………………………….……………………………………………………………………………………

136) 55 y complain of dyspnea, PND with past history of mitral valve disease diagnosis is
a-LT side HF
b- RT side HF
c- pnemothrax
d-P.E

The correct answer is a


……………………………………………………………………………………………………………………………………

137) clonidin is decrease effect of


a- benzotropin (anticholinirgic for Parkinson. Not affected by Clonidin)
b-levo dopa (for Parkinson. Changed in the brain to Dopamen. Clonidin  the
effect of Levodopa through Unknown mechanism)
c-rubstin?????

the correct answer is b


Clonodin is α2 agonist used to TTT hypertension. α2 receptor in the brain cause  of
both COP & peripheral resistant .
138) 2 y baby with gray to green patch in lower back, no redness or hotness, diagnosis
is
a- child abuse
b-no ttt need
c- bleeding tendency
d????

The correct answer is b (I'm not sure )

I think it is " Mongolian spot " , visible in 6 month and normally disappear to 3-5
years. No need treatment.

…………………………………………………………………………………………………………………………………………………

139) 15y boy with unilateral gyncomastia your advice is


a- my resolve spontiniously
b-there is variation from person to person
c-decrease use of soda oil or fish oil
d-???????????

the correct answer is a

- uni- or bilateral gynecomastia occur normally in newborn & at puberty

…………………………………………………………………………………………………………………………………………………

140) 6m baby with mild viral diarrhea , ttt by ORS as


a-100ml/kg for 4 hour then 50 ml/kg /day after
b-50>>>>>>>>>>>>>>>>.50>>>>>>>>>>
c-100>>>>>>>>>>>>>>>>100>>>>>>>>
d-50>>>>>>>>>>>>>>>>>100>>>>>>
141) 36 y female with breast mass mobile and change with menstrual cycle , no skin
dimple or fathering. Your advice is
a-repeat exam after 2 cycle
b-make biopsy
c-fine needle aspiration (there is NO singe of breast cancer. It is Fibroadenoma.
Just do FNA to exclude cancer & relive the pt)
d-oral contraception

The correct answer is a (I'm not sure )

…………………………………………………………………………………………………………………………………………………

142) 50y female with breast cancer and CA125 elevate. So elevation due to
a-breast cancer
b-associate with ovarian cancer
c-due to old age
d-normal variation

The correct answer is a , beause this pt. high risk of breast cancer.

- CA125 is a tumor marker mostly used for ovarian Ca, but it's also used with
endometrial, fallopian, breast, & GIT Ca

…………………………………………………………………………………………………………………………………………………

143) 25y female with bradicardia and palpitation. ECG normal except HR130 and apical
pulse is 210 .past history of full ttt ovarian teratoma, so your advice is
a- struma ovari should be consider
b-vagal stimulate should be done
c- referred to cardiology

The correct answer is a

…………………………………………………………………………………………………………………………………………………

144) pt with alcohol drinked complain of headache , dilated pupil hyperactivity,


agitation .he had history of alcohol withdrawal last week so ttt is
a-diazepam
b-naxtrol
c-haloperidol
d?????????

the correct answer is a

-diazepam (for seizure  given 1st occur in the 1st 12h).


-haloperidol (for hallucination  given 2nd occur in the 1st 2 days).
145) 50y man with chronic psychosis and not complains for ttt .your advice
a- depot haloperidol or floxtin
b-oral lorasepam
c-oral buspiron
d-???????

The correct answer is a

…………………………………………………………………………………………………………………………………………………

146) 15y boy appear patch in rt lower leg these patch is clear center , red in peripheral,
no fever no other complain so diagnosis
a-contact dermatitis
b-tinea corpora
c- lyme disease
d-???????

The correct answer is b

-tinea corpora: worm fungal infection which transmitted by contact skin & has
clinical picture as in the question .

…………………………………………………………………………………………………………………………………………………

147) old pt with of IHD complain for 2 mon of redness in lower leg and pulse dim-
inched in dorsalis pedis these redness increase in dependant position and limp is
cold and no swelling ,diagnosis is:
a-arterial inssuficncy
b-thrmbophibitis
c-cellulites
d??????

the correct answer is a

-thrombophilibitis: present with pain, swelling and redness .


-celluitis: present with redness, hotness and tender.

…………………………………………………………………………………………………………………………………………………

148) pt with heart disease complain of LL ischemia your advice


a-referred to cardiology
b-""""""""""""vascular surgery
c- start heparin
d-??????????/

?????????
149) female after sexual attack on exam hymen tear in
a-2 o'clock
b-4"""""""
c-6""""""
d-8""""""""

the correct answer is c

-tear appear between 5 and 7 o'clock

…………………………………………………………………………………………………………………………………………………

150) 7y boy complain of limb. CT show a vascular necrosis in epiphysis of femur your
advice
a- surgical ttt
b-splint for 6m
c- physiotherapy
d????????

Q not complete: but with these MCQs , the correct answer is b

This case is "perth's diseas" : vascular necrosis of femoral head. 5- 10 years . usually self
limiting with symptoms lasting< 18 months. Present with painless limb , limited
abduction and internal rotation.
Treatment: 1- observation if there is limited femoral head involvement or full ROM .
2- if extensive or if decrease ROM, consider bracing, hip abduction with cast.

…………………………………………………………………………………………………………………………………………………

151) pt with trachoma in eye . for prevention you should


a- water
b- """""""""""""""""+eradication of organism
c- mass ttt
d???????????

the correct answer is a

-trachoma in the eye is a bacterial infection caused by Chlamydia trachomatis


which is transmitted by poor haygen & contaminated H2O. TTT by antibiotic as
erythromycin & Doxycycline. Surgery to prevent scar

…………………………………………………………………………………………………………………………………………………

152) your advice to prevent plaque disease is


a-hand washing
b-rodent eradication
c-spry insect side
d-?????????? ?????
153) pt with severe headache and decrease in visual acuity ,pupil is dilated, so ttt
a-pilocarpin drop and ophthalmology referred
b-ergotamine
c-NSID
d???????

the correct answer is a

- this is closed angle glaucoma which characterized by sudden severe headache,


red eye,  visual acuity, & dilated pupil. Pilocarpin is parasympathomimmic which
help in relive the pain

…………………………………………………………………………………………………………………………………………………

154) main ttt of non inflammatory acne is


a-ritonic acid
b-clindmycin;
c-azalic acid
d-erythromycin

the correct answer is a

-clindamycine & erythromycin are treatment of inflammatory acne.


. –azalic acid : treatment of non-inflammatory & inflammatory acne.
. – ritonic acid : treatment of sever acne & non-inflammatory
…………………………………………………………………………………………………………………………………………………

155) pregnant with insulin dependant with good control, so to decrease risk of
congenital disease
a-good metabolic control before pregnancy
b-"""""""""""""""""""""""1st trimester
c-""""""""""""""""""""""""2nd """""
d-""""""""""""""""""""""""3rd """""""

the correct answer is b(I'm not sure )

…………………………………………………………………………………………………………………………………………………

156) female not married with normal investigation except FBS=142. RBS196. so ttt
a-give insulin subcutaneous
b-advice not become married
c-barrier contraceptive is good
d- BMI control

the correct answer is d


157) diapetic pt come to you with disturbance in conscious RBS : 65. so main drug that
cause hypoglycemia:
a-sulphnylurea
b-bugunid
c-acabos α-glucosidase inhibitor
d-pheniform

the correct answer a

-sulphnylurea ( insulin secretion, so, it can cause hypoglycemia)


-bugunid ( insulin sensitization, so, can not cause hypoglycemia)
-acabos α-glucosidase inhibitor ( used for postprandial hyperglycemia, so, can
. . not cause hypoglycemia)
-pheniform ( insulin sensitization, so, can not cause hypoglycemia)
…………………………………………………………………………………………………………………………………………………

158) 6m boy with fever you should give antipyretic to decrease risk of
a-febrile convulsion
b- epilepsy
c- disseminate bacteria
d???????????????

The correct answer is a

…………………………………………………………………………………………………………………………………………………

159) picture in computer appear vesicle , bulla and erythama in chest skin so ttt
a- acyclovir cream
b-betamethzone cream
c-floclvir
d- erythromycin

multiple choice not clear.

This case is "herpes zoster" . treatment of herpes zoser are antiviral, analgesic.
Antiviral are ( systemic ) and include: acyclovire, famciclovire, acyclovire. In multiple
choice there is "floclvir" and there is not drug have this name. I think the writer Q
written in wrong spelling. I think the choice c is famciclovire and it i s correct
answer. Famciclovir: 500 mg T.I.D for 7 days
160) pt with scale in hair margin and nasal fold and behind ear with papule and irregular
erythema so ttt is
a-nizoral cream
b- atovit
c- acyclovir
d-antibiotic

the correct answer is a ( this is seborrheic dermatitis )

…………………………………………………………………………………………………………………………………………………

161) 14y girl with athralgia and photosensitivity and malar flush. And protinurea so
diagnosis is
a-RA
b-lupus nephritis
c-UTI
d-??????????

The correct answer is b

…………………………………………………………………………………………………………………………………………………

162) question about diarrhea and Yesinia bacteria

Yersiniosis : it is infectious disease caused by Yesinia bacteria. There are 3 types of


Yesinia bacteria : 1- Y.entercolitis , 2- Y.pseudotuberculosis, 3-Y. pestis.
Y.enetercolitis cause bloody diarrhea, terminal ileitis and mesenteric adenitis.
Diagnosis by serological : rise in antibody titer . Treatment : usually self limiting and
tetracycline for sever infection.

…………………………………………………………………………………………………………………………………………………

163) paraplegia pt with ulcer in lower back 2+2 cm and lose of dermis and epidermis
these ulcer in stage
a- I
b-II
c-III
d-IV

the correct answer is b

-stage I : non-blanchable redness that NOT subside after relive of the pressure
-stage II : damage to epidermis & dermis but NOT deeper
-stageIII : subcutaneous tissue involvement
-stageIV : deeper than subcutaneous tissue as muscles & bones
164) PREGNANT LADY prim at labor pain , on exa cervical in stage I of labor so pain
management is
a-morphine IM
b-epidural anesthesia
c-general"""""""""
d-local"""""""

The correct answer is b

…………………………………………………………………………………………………………………………………………………

165) Psycatric pt on antipsychotic drug most drug that lead to impotence with
antipsychotic is
a- proprnlol
b-NSAI
c-ACEI
d-?????????

the correct answer is a


…………………………………………………………………………………………………………………………………………………

166) man present with painless ulcer in his penis with indurate base and everted edge
so diagnosis is
a- syphilis
b- gonorrhea
c- choncroid
d- HSV

The correct answer is a

- syphilis : painless ulcer in the penis


- gonorrhea: STD by bacterial infection cause pain & discharge but NO ulcer
- choncroid: STD by bacterial infection cause pain & discharge but NO ulcer
- HSV : STD by virus that cause painful ulcer with discharge.
…………………………………………………………………………………………………………………………………………………

167) man have long history of urethral stricture present with tender right testis and
WBC in urine so diagnosis is
a-epddimorchitis
b- testicular torchin
c- varicosel
d-??????

the correct answer is a


-epddimorchitis : occur with UTI & urinary retension)
- testicular torchin : sudden acute severe testicular pain with elevated
. transversely ling testes)
- varicosel : due to  intra-abdomenal pressure.
168) man use saldinafil (Viagra), to prevent hypotension you should not use
a-nitrate
b-B blocker
c-ACIE
d-CCB

the correct answer is a

…………………………………………………………………………………………………………………………………………………

169) female complain of painless odorless and colorless vaginal discharge that appear
after intercourse so ttt
a-give antibiotic
b-douche after intercourse
c- cervical cancer should be consider
d-may be due to chronic salpingitis

the correct answer is b

Color Smell Pain OR Itching Other


 during
Normal vaginal
Clear OR milky Odorless NO ovulation, sex,
discharge
breastfeeding
Bacterial White-gray OR
Fishy Yes  during sex
vaginosis yellow
Watery Or Yes especially
Trichomoniasis
yellow during urination
White-cheesy Yes especially
Candida
thick sticky during sex
…………………………………………………………………………………………………………………………………………………

170) UTI>14 day, most probably cause pylonphritis


a-,05%
b-,5%
c-5%
d-50%

Difficult Q , This Q referral to urologist.

…………………………………………………………………………………………………………………………………………………

171) patient complain of right iliac fossa mass so diagnosis


a-diverticulitis
b-appendicitis
c-pancrtitis
d-chrons disease
- the correct answer is d
172) pt with long history of UC on endoscopes see polyp and cancer lesion on left colon
so ttt
a-ttt of anemia
b-left hemicolctomy
c-total colctomy
d- remove polyp

…………………………………………………………………………………………………………………………………………………

173) female with hair on different site of body and refuse intake of food and BMI<18
and feel as body is fat so diagnosis
a-anorexia nervosa
b-bulimia nervosa
c-body dismorphic syndrome
d- anxiety

the correct answer is a

-anorexia nervosa: decrease body wt. amenorrhea and lanugo (hair ).


-bulimia nervosa: normal or increase body wt. restrict eating following by
overeating then guilt.

…………………………………………………………………………………………………………………………………………………

174) obese female, insulin resistance and hairstisim so diagnosis


a-poly cystic ovary
b-hyperprolctinmia
c-familial
d-?"????????

the correct answer is a

…………………………………………………………………………………………………………………………………………………

175) boy 3 day after flue symptom develop conjunctivitis with occipital and neck L.N
enlarged so diagnosis is
a-adenoviruses
b-streptococcus
c-HSV
D??????????

the correct answer is a

…………………………………………………………………………………………………………………………………………………

176) child with asthma use betamethazone, most common side effect is
a-increase intraocular pressure
b-epilepsy
c-growth retardation
d-?????????????
. the correct answer is c
177) curve of HBV marker

answered in Q 38

…………………………………………………………………………………………………………………………………………………

178) sickling pt after acute attack , discharge on


a- penicillin
b-iron
c-vitamin
d???????????

the correct answer is a

prophylactic of pneumococcal infection by vaccination and oral penicillin

…………………………………………………………………………………………………………………………………………………

179) 6m with cough and wheezy chest .diagnosis is


a- asthma
b- broncholitis
c-pneumonia
d-F.B aspiration

The correct answer is b

- asthma : after 2 years old)


- broncholitis : before 2 years old)
-pneumonia : associated with crypitation
-F.B aspiration : sudden wheezing

…………………………………………………………………………………………………………………………………………………

180) 15y old with pilonidal sinuse so ttt


a-incision surgery
b- local antibiotic
c-daily clean
d-?????????

The correct answer is c


. pilonidal sinus : sinus tract witch commonly contain hairs. Treatment , firstly
conservative ttt in mild case remove all hair, washing cleaning . if not relive: surgery
181) Female pt 8 wks postpartum,not smoker diagnosed to have asthma,her asthma
was not controlled she attended ER 3 times last month,on B2 agonist and oral
steroid,she came c/o wheezing and s.o.b mildly cyanosed using her intercostal
muscles,wheezy chest,BP:160/100 P:120 PO2:72 PEF:36,there is oedema in her
foot up to the knee,the most likely diagnosis:
a. COPD
b. pulmonary embolism
c. Acute asthma attack
d. Angioedema

The correct answer is a or c

……………………………………………………………………………………………………………………………………………………

182) Female pt developed sudden loss of vision(both eyes) while she was walking
down the street,also c/o numbness and tingling jn her feet ,there is discrepancy
b/w the complaint and the finding,
O/E reflexes and ankle jerks preserved,there is decrease in the sensation and
weakness in the lower muscles not going with the anatomy,what is your action:

a. Call ophthalmologist
b. Call neurologist
c. call psychiatrist
d. reassure her and ask her about the stressors
the correct answer is : d

……………………………………………………………………………………………….........................................................

183) same scenario in Q.182 what is the diagnosis:

a. Conversion disorder
b. Somatoform disorder

The correct answer is a

1 – somatization disorder: female before age 30 years . symptoms include: .


. two GIT , four site of pain , one sexual dysfunction , one pseudoneuron .
. 2- conversion disorder: symptoms include voluntary or sensory .
. 3- hypochondriasis: fear from life threatening disease.
. 4- body dysmorphic disorder: aware from his imaging.
. 5- somatoform pain disorder: intensity pain is main symptoms
184) male pt developed corneal ulcer in his Rt eye after trauma what is the Mx:
a. Antibiotic and cycloplagia is mydrasis and refer to ophthalmology
b. topical steroid

the correct answer is a

Because infection is a common occurrence in corneal ulcers, your ophthalmologist


will prescribe antibiotic eyedrops. If the infection appears very large, you may need
to use these drops as often as one drop an hour. -Oral pain medications will be
prescribed to control the pain. Pain can also be controlled with special eyedrops
that keep your pupil dilated (Anticholinergics such as atropine, hyoscyamine, and
scopolamine)

...........................................................................................................................................................

185) female pt with Rt eye pain and redness with watery discharge,no h.o
trauma,itching,O/E there is diffuse congestion in the conjunctiva and watery
discharge what you'll do:
a. give Ab
b. give antihistamine
c. topical steroid
d. refer her to the ophthalmologist

???????????????

……………………………………………………………………………………………………………………………………………………..

186) Epidemic disease in poor sanitation areas affecting children and young adults:
a. hep A
b. B
c. C
d. D

The correct answer is a


187) mths baby with crying episodes+current jelly stool,looks slightly pale,signs of
obstruction wht is your Mx:
a. barium enema
b. immediate surgery
c. I.v fluid & wait for resolution

The correct answer is a

Intussusceptions :

Condition in which one portion of bowel invaginates into an adjacent segment


"usually proximal to ileocecal valve" . most common in first two years of life "
usually between 3 months and 3 years of life" . Abrupt –onest , colicky abdominal
pain , often accompanied by flexed knee and vomiting , ( one-off pain ) child may
appear well between episodes. Classic triad : abdominal pain , vomiting , blood per
rectum" only one third of pt." . Late signs : bloody mucus in stool "currant jelly
stool" , abdominal tenderness , palpable "sausage- shape " . RUQ abdominal mas.

Investigation & treatment :


-correct any volume and electrolyte abnormality and check ( cbc)
Abdominal film may be normal in early stage , and see obstruction , perforation in
late stage, US see " target sign"
In setting of high clinical suspicion >>> air-contrast barium enema, should be
performed without delay. As diagnostic in 95% of cases and curative in 80% of cases
perform surgical reduction of gangrenous bowel.

……………………………………………………………………………………………………………………….………………………......

188) 17 y.o adolescent, athletic ,with h/o Rt foot pain planter surface,diagnosis is:
a. planter fasciaitis
b. valux……
c. valux…..

Q not complete.

Planter fasciitis( heel spur syndrome ) :

Repetitive strain injury causing micro tears and inflammation of planter fascia.
Common in athletes , also associated with D.M , obesity , seronegative and
seropositive arthritis.
Clinical feature: morning pain and stiffiness , intense pain when walking from rest
and that subsides as pt. continuous to walk. Swelling and tenderness over sole,
greatest at medical calcaneal tubercle and 1-2 cm distal to along planter fascia .
pain with toe dorsiflexion
189) pregnant lady 16 wks presented with vaginal bleeding ,enlarged
abdomen,vomiting ,her uterus is smaller than expected for the gestational
age,BhCG 80,U/S snowstorm appearance,diagnosis:
a. complete hydatiform mole
b. partial hydatiform mole

The correct answer is a

………………………………………………………………………………………………………………………………………………….....

190) 10. 12 y.o boy c/o abdominal pain after playing football, he denied any h/o
trauma, the pain is in the Lt paraumbilical region what inx you want to do:
a. CXR
b. ultrasound kidney

Q not complete

………………………………………………………………………………………………………………………………………………….....

191) 5 y.o child with h.o fever and swelling of the face ant to the both ears (parotid
gland enlargement) what is the most common complication:
a. Labrynthitis
b. meningitis
c. encephalitis
d. orchitis

The correct answer is b

- most common complication of mumps after puberty . but it is rare occur pre-
puberty.

…………………………………………………………………………………………………………………………………………………

192) . what is the meaning of difficulty breathing:


a. dyspnia
b. tachycardia

The correct answer is a


193) A female patient on the 3rd week postpartum. She says to the physician that the
frequently visualizes snakes crawling to her baby’s bed. She knows that it is
impossible but she cannot remove the idea from her head. She says she wakes up
around 50 times at night to check her baby. This problem prevents her from
getting good sleep and it started to affect her marriage. What is this problem she
is experiencing?
a. An obsession
b. A hallucination
c. A postpartum psychosis
d. A Delusion

The correct answer is a ( 100 % correct  )

-obsession : persistent, unwanted, and intrusive ideas , thoughts, impulses or images


-hallucination: perception of objective or event without existing external stimulus
-illusion: false perception of actual external stimulus.
- delusion: fixed false idiosyncratic belief.

………………………………………………………………………………………………………………………….………………………….

194) female pt c/o sever migraine that affecting her work,she mentioned that she was
improved in her last pregnancy,to prevent that:
a. biofeedback
b. propranolol

The correct answer is b

- migraine prophylactic :
1- b-blocker (propranolol) : first line treatment (note: contindication in pregnancy)
2- TCA (amitriptyline )
3- anticovulsant
4- calcium channel blocker

……………………………………………………………………………………………………………………………………………………..

195) About DM in KSA:


a.. most of NIDDM are obese ( correct )

…………………………………………………………………………………………………………………………………………………….

196) Flu vaccine not given to the baby who is allergic to:
a. egg

contraindication of flu vaccine are egg allergic and hypersensitivity to thimerosal


197) 17. Pt with asymptomatic Trichomniasis:
a. treat her anyway regardless
b. treat her if she is symptomatic only

The correct answer is a

-treatment of trichomnias :

Single dose of metradinazole for symptomatic and asymptomatic pt. and


treatment partner ( because it is sexual transmitted disease )
But not treatment of asymptomatic trichomniasis in first trimester of
pregnancy. (contraindication in pregnancy )

…………………………………………………………………………………………………………………………..…………………………

198) 7 y.o,she missed her second dose of varecila vaccine,the first one about 1 y ago
what you'll do:
a. give her double dose vaccine
b. give her the second dose only
c. see if she has antibody and act accordingly

The correct answer is b

…………………………………………………………………………………………………………………………………………………….

199) pt with gonorrhea infection what else you want to check for
a. Clamydia trachomatis

…………………………………………………………………………………………………………………………………………………….

200) female pt with Aortic stenosis,she developed syncope while she was in the class
and she recovered immediately,what is the cause of syncope:
a. valvular rupture
b. systemic hypotension

The correct answer is b

……………………………………………………………………………………………………………………………………………………..

201) diabetic pt well controlled,she came with h.o dizziness and sweating after taking
a medication BS:60 what drug that cause her prob:

-Sulfanyurea can cause this side effects.


202) male pt with acute urine retention what is your action:
a. insert folly's cath and ask him to come back to the clinic

…………………………………………………………………………………………………………………………………………………….

203) In battered women which is true:


a. mostly they come from poor socioeconomic area
b. usually they marry a second violent man
c. mostly they come to the E/R c/o………..
d. mostly they think that the husband respond like this because they still have
. strong feeling for them

Answer ( d )

Battered women is Women who are physically and mentally abused over an
extended period, usually by a husband or other dominant male figure.
Characteristics of the battered woman syndrome are helplessness, constant fear,
and a perceived inability to escape. So I think choice d is correct answer

……………………………………………………………………………………………………………………………………………………..

204) smoking withdrawal symptoms peak at:


a. 1-2 days
b. 2-4 days
c. 5.7 days
d. 10-14 days

The correct answer is a

……………………………………………………………………………………………………………………….…………………………….

205) Mother who is breast feeding and she want to take MMR vaccine what is your
advice:
a. can be given safely during lactation
b. contain live bacteria that will be transmitted to the baby
c. stop breast feeding for 72 hrs after taking the vaccine

The correct answer is a

MMR : contraindication during pregnancy , and women should be avoid


pregnant in 4 weeks followed MMR vaccine . MMR is safe during lactation.
206) male pt c/o pain in his Rt elbow,he said that he is using the hummer a lot in his
work diagnosis:
a. lateral epichondylitis

-Lateral epicondylitis( inflammation of common extensor tendon ) also known as (tennis


elbow, shooter's elbow and archer's elbow is a condition where the outer part of the
elbow becomes sore and tender. It is commonly associated with playing tennis and
racquet sports.
-Medial epicondylitis (inflammation of common flexor elbow) also know (golfer elbow)

……………………………………………………………………………………………………………………………………………………

207) 50 y.o male with difficulty swallowing food with wt loss:


1. Oesophageal cancer

question not complete . but most likely is cancer.


causes of dysphagia food than liquid are :
1- carcinoma … 2- stricture … 3- plummer vision syndrome = web ( iron deficieny
anemia + golssitis )

……………………………………………………………………………………………………………………………………………………

208) . young female with pain in her elbow(lateral epichondylitis) best treatment is :
a. ………….+NSAID
b. electric ………….

question not complete .

- treatment of lateral epichondylitis:


1- first line : NSAID +rest + ice
2- second line : corticosteroid injection
3- third line : surgery .percutaneous release of common tendon

………………………………………………………………………………………………………………………………..…………………..

209) what drug that improve the survival in CHF


1. digoxin
2. Hydralazin

question is not complete .

Improve mortaility in CHF : B-blocker + ACE + ARB ( most important is ACE )


210) old man with bilateral knee pain and tenderness that increase with walking and
relieved by rest;
a. RA
b. OA

The correct answer is b

OA: pain with activity and weight bearing . and improve with rest .
RA: morning stiffness > 1 hour . painful and warm swelling of multiple symmetric
joint .

……………………………………………………………………………………………………………………………………………………

211) Regarding peritonitis:


a.Complicated appendectomy the cause is anerobe organism
b. rigidity and the cause is paralytic ileus
c. can be caused by chemical erosions
d. …………..

The correct answer is c

……………………………………………………………………………………………………………………………………………………..

212) regarding MI all of the following true except:


a. unstable angina,longer duration of pain and can occur even at rest.
b. stable angina,shorter duration and occur with excertion
c. there should be q wave in MI
d. even if there is very painful unstable angina the cardiac enzymes will be normal
e. ………………

all of the above are correct ???

……………………………………………………………………………………………………………………………………………………

213) Pt with scoliosis, you need to refer him to the ortho when the degree is:
a. 5
b. 10
c. 15
d. 20

The correct answer is d


214) 6 mths baby with undescending testis which is true:
a. till the mother that he need syrgery
b. in most of the cases spontaneous descent after 1 year
c. surgery indicated when he is 4 years
d. unlikely to become malignant

The correct answer is a

-after 1 year testes not desending. And surgery(orchiopexy) do from 6 –18months

……………………………………………………………………………………………………………………………………………………

215) 24 y.o Pt with asymptomatic congenital inguinal hernia:


a. immediate surgery
b. surgery indicated when he is >35 y
c. elective surgery if it is reducible
d. …………..

The correct answer is c


…………………………………………………………………………………………………………………………………………………..

216) the most effective thing regarding counseling:


a. family rapport
b. well adjusted appointment before counseling
c. ……………….

?????

…………………………………………………………………………………………………………………………………………………..
217) 5 years old C/O limping in CT there is a venous necrosis ttt is:
a. surgery
b. splent
c. physiotherpy

Q not complete:

This case is "perth's diseas" : vascular necrosis of femoral head. 5- 10


years . usually self limiting with symptoms lasting< 18 months. Present
with painless limb , limited abduction and internal rotation.
Treatment: 1- observation if there is limited femoral head involvement
or full ROM . 2- if extensive or if decrease ROM, consider bracing, hip
abduction with cast.
218) Pt. has DM and renal impairment when he had diabetic nephropathy:there is
curve for albumin
a. 5y
b. 10y
c. 20y
d. 25y

……………………………………………………………………………………………………………………………………

219) Pt has alzahimar disease and halusination and delusion ttt:


a. Halopridole

Psychotic symptoms (e.g. hallucinations and delusions), agitation and aggressive


behavior are common in patients with Alzheimer’s Disease. A study suggests
that haloperidol at a dose of 2-3 mgs/day is effective and well tolerated by most
patients.
220) Generalize anexity disorder best ttt:
a. SSRIs
b. tricyclic A D

The correct answer is a

……………………………………………………………………………………………………………………………………

221) Major depression management:


a. Intial therapy even sever

 Management of major depression disorder:


1-pharmacotherapy: effective in 50 – 70% .allow for 2-6 weeks to take effect , treat
more than 6 months ( SSRI, TCAs, MAOIs).
2-psycotherapy: psychotherapy combined with antidepressant is more effect than
either treatment alone.
3- Electroconvulsion ( ECCT ).
4- phototherapy: effective for pt. who has a seasonal pattern.

……………………………………………………………………………………………………………………………………

222) Psychiatric pt with un compliance of drugs ttt:


a. depro halopredol

……………………………………………………………………………………………………………………………………

223) Strongest risk factor for strok:


a. Hypertension
b. Atrial fibrillation

The correct answer is a


224) Pt. with salpingitis and there is swelling in pelvis in posterior fornex and it is
fluctuant m:
a. Colpotomy
b. Laproscopic

The correct answer is a (I'm not sure )

colpotomy, also known as a vaginotomy, is a procedure by which an incision is


made in the vagina.
Purpose: A colpotomy is performed either to visualize pelvic structures or to
perform surgery on the fallopian tubes or ovaries.

……………………………………………………………………………………………………………………………………

225) Child swallowing battery in the oesophegus, management?


a. broncoscoby
226) In the appendisits the histology is:
a. leukocyte in muscle
b. layer of lymphoid
c. tumor
d. plasma cell

the correct answer is a

in appendicitis : neutrophil exudation throughout mucus, submucus, and


muscularis

……………………………………………………………………………………………………………………………………

227) water in the body:


a. 40%
b. differ depend on age and sex

the correct answer is b

Guyton's Textbook of Medical Physiology states that "the total amount of water in
a man of average weight (70 kilograms) is approximately 40 liters, averaging 57% of
his total body weight. In a newborn infant, this may be as high as 75% of the body
weight, but it progressively decreases from birth to old age, most of the decrease
occurring during the first 10 years of life. Also, obesity decreases the percentage of
water in the body, sometimes to as low as 45%".

……………………………………………………………………………………………………………………………………

228) Corneal ulcer ttt:


a. Antibiotic and cycloplagia is mydrasis and refer to ophthalmology

-Because infection is a common occurrence in corneal ulcers, your ophthalmologist


will prescribe antibiotic eyedrops. If the infection appears very large, you may need
to use these drops as often as one drop an hour. -Oral pain medications will be
prescribed to control the pain. Pain can also be controlled with special eyedrops
that keep your pupil dilated (Anticholinergics such as atropine, hyoscyamine, and
scopolamine)
229) regarding drainage of the abscess one of the following is true:
a. Carbuncle and frunculosis need drainage
b. Usually give ciphtriaxon and penicillin post draing

the correct answer is a

Frunculosis and carbuncles are pus-filled infected lumps on the skin. They usually
occur as a one-off in a healthy person. Treatment commonly involves draining the
pus and taking a course of antibiotics. If you have recurring boils you may be
advised to have tests to check for an underlying cause.

……………………………………………………………………………………………………………………………………

230) Salpingitis and PID on penicillin but not improve the most likely organism is :
a. Chlamydia trachomatis
b. Neisseria gonorrhoeae

The correct answer is b

Empiric antibiotic regimens should be aimed at treating likely causative agents, that
is, N. gonorrhoeae, C. trachomatis, genital mycoplasmas, and bacterial vaginosis-
associated endogenous microflora. The latter include anaerobic (Bacteroides and
Prevotella species and anaerobic streptococci) as well as aerobic organisms ( G.
vaginalis, E. coli, and facultative streptococci). Except for N. gonorrhoeae and some
anaerobes, resistance is not yet a clinical problem.

……………………………………………………………………………………………………………………………………

231) Wound at end inflammatory phase when:


a. Epithelial tissue formation
b. Angiogenisis
c. when the wound clean
d. Scar formation

The correct answer is b


232) Juvenile RA ttt:
a. Aspirine
b. Steroid
c. Penicillamine
d. dydrocloroquin

The correct answer is a

The treatment of JRA focuses on suppressing inflammation, preserving and


maximizing function, preventing deformity, and preventing blindness. NSAIDs are
the first choice in the treatment of JRA.

……………………………………………………………………………………………………………………………………

233) Pt. has hemorrhoid with 4 degree ttt:


a. Hemoridectomy
b. band ligation
c. sclerotherapy
d. fiber diet
-The correct answer is a.

- Classification of internal hemorrhoid :


First degree : hemorrhoid do not prolapsed
Second degree : hemorrhoid prolapsed upon defecation but spontaneously reduce
Third degree : hemorrhoid prolapsed upon defecation , but must be manually reduce
Fourth degree : hemorrhoids are prolapsed and cannot be manually reduce

-treatment:
First & Second : life style modification ( dite)
Third : life style modification with (band ligation, sclerotherapy or cryotherapy ) , if
Failed go to surgery.
Fourth : surgery ( hemorrhiodectomy )
234) Pt. come with history of tinia capitis ttt:
a. tar shampoo
b. Fluconazol

The correct answer is b

Newer antifungal medications, such as ketoconazole, itraconazole,


terbinafine, and fluconazole, have been reported as effective alternative
therapeutic agents for tinea capitis. Of these agents, itraconazole and
terbinafine are used most commonly.

……………………………………………………………………………………………………………………………………

235) Pt. with history of COPD the most action to prevent complication is:
a. Pnemoccoccal vaccine
b. Smoking stop
c. Oral steroid
d. B2 agonist

The correct answer is b or a

……………………………………………………………………………………………………………………………………
236) Pt. with congenital hip dislocation :
a. abducting at flexed hip can causes click or tali

Congenital hip dislocation (CHD) occurs , more commonly in girls than in boys. The left hip is
twice as often involved as the right and bilateral dislocation occurs in more than 25 percent of
affected children. The criteria for the diagnosis of congenital dislocation of the hip include both
physical and radiographic findings. Certain clinical signs have been identified that are helpful in
the evaluation of newborns and infants for possible CHD, which include the following:

 limited abduction of the flexed hip, due to shortening and contraction of the hip
adductors;
 increase in depth or asymmetry of the inguinal or thigh skinfolds;
 shortening of one leg;
 Allis' or Galeazzi's sign -- lower position of knee of the affected side when knees and hips
are flexed, due to location of femoral head posterior to acetabulum in this position;
 Ortolani's "jerk" sign ("clunk of entry" or reduction sign);
 Barlow's test ("clunk of exit" or dislocation sign);
 telescoping or pistoning action of thighs, due to lack of containment of femoral head
with acetabulum;
 Trendelenburg's test -- drop of normal hip when child, standing on both feet, elevates
unaffected limb and bears weight on affected side, due to weakness of hip abductors;
 waddling type of gait.

There are characteristic radiographic presentations which are present in each of the stages of
CHD. The term congenital hip dysplasia generally refers to delayed or defective development of
the hip joint leading to a deranged articular relationship between an abnormal acetabulum and
a deformed proximal end of the femur. This condition is considered a precursor of the clinical
entities of subluxation and dislocation of the hip. In congenital subluxation of the hip, there is
an abnormal relationship between the femoral head and acetabulum, but the two are in
contact. Congenital dislocation of the hip, on the other hand, is associated with a complete loss
of contact of the femoral head with the acetabular cartilage. Unfortunately, the femoral head
and acetabulum in the newborn cannot be assessed by direct visualization, since the fe moral
head is not ossified and is a cartilaginous body which is not visualized on plain films. The
ossification center for the femoral head generally appears between three and six months; a
delay in its appearance is an indication of congenital hip dysplasia. The neck of the femur is used
for ascertaining the relationship between the acetabulum and the femoral head until the
ossification center appears. The measurement used to evaluate the relationship of the femoral
head and acetabulum include Hilgenreiner line, acetabular index, Perkin-Ombredanne line,
Shenton-Menard line, and Andren-von Rosen line.

The principal treatment for CHD is conservative, especially if diagnosed early. The most common
technique is to reduce the dislocation of the femoral head by means of a flexion/abduction
maneuver, for a sufficient period of time to permit proper growth of the head and acetabulum,
which in turn assures a congruent and stable hip joint. This technique is usually performed on
patients in the very early stages of CHD and in infants under two years of age; which include
splinting, with a Frejka splint or Pavlik harness. Colonna or Buck's skin traction is used in children
2 to 12 years of age, with a well-padded spica cast applied simultaneously to the unaffected side
237) Colon cancer with stage 3 give the chemotherapy:
a. As soon as possible
b. 1 month

The correct answer is a (I'm not sure because MCQs not complete )

Treatment for Stage 3 Colon Cancer

Treatment for stage 3 colon cancer generally consists of a surgical resection


followed by chemotherapy. In a surgical resection, a surgeon removes the part of
the colon affected by the tumor and joins the remaining healthy sections together
to form one long, healthy piece.

The standard chemotherapy regimen used to be six months of treatment with 5-FU
and leucovorin, but that "cocktail" was developed decades ago and is seldom used
anymore. there are many new chemotherapy regimens available for stage 3 colon
cancer.

……………………………………………………………………………………………………………………………………

238) 56 y old present with vasomotor rhinitis


a. Local anti histamine
b. Local decongestion
c. Local steroid
d. Systemic antibiotic

The correct answer is b

Vasomotor rhinitis is a nonallergic condition that causes a constant runny nose,


sneezing, and nasal congestion.

……………………………………………………………………………………………………………………………………

239) Sickle Cell Anemia give prophlaxis:


a. Penicillin
b. Iron
The correct answer is a

TREATMENT
■ Acute crisis: Analgesia and hydration.
■ Hydroxyurea to the amount of fetal hemoglobin.
■ H. influenzae and pneumococcal vaccines; prophylactic penicillin for
Children 5 years of age.
■ Acute chest syndrome: Respiratory support and exchange transfusion.
240) diagnosis of thalasimia minor:
a. Hb A2 and Hb f
b. Microcytosis

the correct answer is a

……………………………………………………………………………………………………………………………………

241) Pt. with MCV decrease and reticulocyt decrease iron deficiency anaemia
investigation:
a. Ferritin level and TIBC and serum iron

……………………………………………………………………………………………………………………………………

242) Born BCG


a. 1month hepatitis b oral polio dpt
b. 2month s
c. 3month s
d. 9 to 12 month mmr
????? I can't understand th Q ( may the writer Q missed some information )
243) E.histolytica cyst is destroyed by:

a. Freezing

b. Boiling

c. Iodine treatment

d. Chlorine

The correct answer is b

Amebiasis (or amoebiasis) is the name of the infection caused by E. histolytica.


To help prevent infection:
• Avoid raw vegetables when in endemic areas as they may have been fertilized
using human feces.
• Boil water or treat with iodine tablets
……………………………………………………………………………………………………………………………………………………

244) Patient after accident, the left ribcage move inward during inspiration and
outward during expiration:

a. Flial chest

Flail chest is a clinical anatomic diagnosis noted in blunt trauma patients with
paradoxical or reverse motion of a chest wall segment while spontaneously
breathing. This clinical finding disappears after intubation with positive pressure
ventilation, which occasionally results in a delayed diagnosis of the condition.

……………………………………………………………………………………………………………………………………………………

245) Greatest risk of stroke:

a. DM

b. Elevated blood pressure

c. Family history of stroke

d. Hyperlipedemia

e. Smooking

the correct answer is b


246) Child has pallor , eats little meat , by investigation :microcytic hypochromic
anemia . what will you do:

a. Trial of iron therary

b. Multivitamin with iron daily

the correct answer is a ( I'm not sure vecause Q is not complete )

……………………………………………………………………………………………………………………………………………………

247) Treatment of mania that does not cause hepatotoxicity

a. Lithium
b. carbamazepine
c. valporic acid
d. lamotrigine

The correct anser is a

Treatment of mood disorder :

Lithium : hepatotoxicity
carbamezapine : agrnulocytosis
valporic asid : neural tube defect

……………………………………………………………………………………………………………………………………………………

248) Sickle cell anemia patient , the macula is cherry red , and absence of afferent
papillary light reflex
a. Retinal artery occlusion.
. b. veine artery occlusion

the correct answer is a , cherry red spot is sign of retinal atery occlusion.
249) Inflammatory bowel disease is idiopathic but one of following is possible
underlying cause

a. Immunological

Inflammatory Bowel Disease Causes

Researchers do not yet know what causes inflammatory bowel disease. Therefore,
IBD is called an idiopathic disease (disease with an unknown cause).

An unknown factor/agent (or a combination of factors) triggers the body’s immune


system to produce an inflammatory reaction in the intestinal tract that continues
without control. As a result of the inflammatory reaction, the intestinal wall is
damaged leading to bloody diarrhea and abdominal pain.

……………………………………………………………………………………………………………………………………………………

250) Patient present with high blood pressure (systolic 200) , tachycardia , mydriasis ,
sweating . what is the toxicity:

a. Anticholenergic

b. Sympathomimetic

c. Tricyclic antidepressant

d. Organophosphorous compounds

the correct answer b

Uses of Sympathomimetics: To treat and prevent reversible bronchospasm


associated with bronchial and nocturnal (nighttime) asthma, chronic
bronchitis,emphysema, exercise-included bronchospasm and other obstructive
airway disease of the lungs.

To treat serious allegoric reactions (epinephrine injection only)


251) Treatment of Chlamydia with pregnancy:

a. Azithromycin

b. Erythromycin base

the correct answer b

For treatment of chlamydia during pregnancy, the CDC recommends:


-erythromycin base, 500 mg orally, four times a day for seven days; or amoxicillin
(Amoxil), 500 mg, three times a day for seven days.
Alternatives include:
-Erythromycin base 250 mg, four times a day for 14 days;
-erythromycin ethylsuccinate 800 mg orally, four times a day for seven days; or
-erythromycin ethylsuccinate 400 mg orally, four times a day for 14 days.

……………………………………………………………………………………………………………………………………………………

252) the maximum body lenght will be reached after menarch by

a -6 months

b. 1 year

A child will have also reached her final adult height about two years after menarche.

……………………………………………………………………………………………………………………………………………………

253) Patient developed sudden loss of vision bilaterally while she was walking in the
street , followed by numbness , the subjective symptoms are different from
objective , and does not match anatomical , what is your diagnosis:

a. Conversion syndrome

Conversion disorder:
Symptoms or deficits of voluntary motor or sensory function (e.g., blindness,
seizure) suggest a condition incompatible with medical processes. Close temporal
relationship to stress or intense emotion. More common in young females and in
lower socioeconomic and less educated groups.
……………………………………………………………………………………………………………………………………………………

254) After inflammatory phase of wound , there will be wound healing by:
a. If the wound is clean
b. Angiogenesis
c. Epithelial tissue
. the correct answer is b
255) Old male with tender knee , pain , crepitus . the diagnosis:

a. Osteorhritis
b. Ankylosin spondylitis
c. Rheumatoid

the correct answer a

Osteorhritis
OARTHRITIS (OA)
■ A chronic, noninflammatory arthritis of movable joints (e.g., DIP joints).
Characterized by deterioration of the articular cartilage and osteophyte
formation at joint surfaces.
■ Risk factors include a _ family history, obesity, and a history of joint
trauma.
■ Hx/PE: Crepitus; decrease range of motion (ROM); pain that worsens with
activity
and weight bearing but improves with rest.

……………………………………………………………………………………………………………………………………………………

256) Mother has baby with cleft palate and asks you what is the chance of having a
second baby with cleft palate or cleft lip ,

a. 25%
b. 50%
c. %1
d. 4%

the correct answer is d

……………………………………………………………………………………………………………………………………………………-

257) 1 liter fluid deficit equals :


. a.1 kg
. Liter of fluid deficit equal 1 kg of fluid as hydration protocol
…………………………….……………………………………………………………………………………………………....................

258) After accident patient with tachycardia, hypotension, what will be your initial
step:
a. Rapid IVF crystalloid
b. CT
Q not complete . management ABC then rapid IVF crystalloid.
259) 6 years child was born to HBS positive mother is HBS positive , he was only
vaccinated by BCG after birth , what you will give him now :
a. HBV + oral polio + DTP + hib
b. HBV + oral polio + dt + MMR +hib
c. HBV + oral polio + Dt + MMR

?????????? all the above are wrong

……………………………………………………………………………………………………………………………………………………

260) Treatment of non inflammatory acne

a. Retinoic acid

Retinoids: Medicines structurally similar to vitamin A are useful in preventing


several types of acne lesions. Topical retinoids are effective in treating the
noninflammatory types of acne (blackheads and whiteheads).

……………………………………………………………………………………………………………………………………………………

261) Treatment of comedones:

a. Topical retinoids.

Comedones: The plural of comedo, the primary sign of acne, consisting of a dilated
(widened) hair follicle filled with keratin squamae (skin debris), bacteria, and sebum
(oil). Comedones may be closed or open.

……………………………………………………………………………………………………………………………………………………

262) Treatment of papules or pustules:

a.Topical benzoyl
b.Peroxide plus topical antibiotics, mainly clindamycin or erythromycin.
c. In severe cases, intralesional steroid injection or oral antibiotics, such as
tetracycline or erythromycin may be added.

The correct answer b

TREATMENT
■ Mild acne: Topical clindamycin or erythromycin; benzoyl peroxide; topical
retinoids.
■ Moderate acne: The above regimen plus oral antibiotics such as tetracycline.
■ Severe nodulocystic acne: Oral isotretinoin (Accutane).
263) Which is not true In emergency management of stroke

a.Give IVF to avoid D5 50%


b.Give diazepam in convulsions .
c.Anticonvulsants not needed in if seizures
d.-Must correct electrolytes
e. Treat elevated blood pressure

the correct answer is c

……………………………………………………………………………………………………………………………………………………

264) SCA complications in adults

a. Cerebral infarction
b. Cerebral hemorrhage

Neurological complications occur in 25% of patients, with transient ischaemic attacks, fits,
cerebral infarction, cerebral haemorrhage and coma. Strokes occur in about
11% of patients under 20 years of age. The most common finding is obstruction of a distal
intracranial internal carotid artery or a proximal middle cerebral artery. 10% of children
without neurological signs or symptoms have abnormal blood-flow velocity indicative of
clinically significant arterial stenosis; such patients have very high risk of stroke. It has now been
demonstrated that if children with stenotic cranial artery lesions, as demonstrated on
transcranial Doppler ultrasonography, are maintained on a regular programme of transfusion
that is designed to suppress erythropoiesis so that no more than 30% of the circulating red cells
are their own, about 90% of strokes in such children could be prevented.
……………………………………………………………………………………………………………………………………………………

265) The most common risk for intracerebral stroke

a. Hypertension
266) The antidepressant used for secondary depression that cause sexual dysfunction

a.Sertatline (SSRIs)
b. Imapiramine
c. Levofluxine

the correct answer a

……………………………………………………………………………………………………………………………………………………

267) Previously healthy female patient presented to ER with dysnea , anexiety ,


tremor , and she breath heavily , the symptoms began 20 minutes before she
came to ER , in the hospital she developed numbness periorbital and in her
fingers , what you will do

a. Ask her to breath into a bag


. b. Take blood sample to look for alcohol toxicity

MCQs not complete

……………………………………………………………………………………………………………………………………………………

268) What is the most important in councling

a. Exclude physical illness

b. Establishing rabbot

c. Family

d. Schedule appointement

?????
269) In breaking bad news

a. Find out how much the patient know


b. Find out how much the patient wants to know

the correct answer a

Robert Buckman's Six Step Protocol for Breaking Bad News

1. Getting started.
2. Finding out how much the patient knows.
3. Finding out how much the patient wants to know.
4. Sharing the information.
5. Responding to the patients feelings.
6. Planning and follow-through.

…………………………………………………………………………………………………………………………………………………

270) Patient with chest pain that aggrevated by couphing, there is added sound on left
sternal border .in ecg you will find

a. St changes

b. Pr prolongation

c. Hypervoltage

the correct answer a . (percarditis)

…………………………………………………………………………………………………………………………………………………

271) The most common site for visceral hemangioma is

a. Liver ( most common site for visceral hemangioma )

A hepatic hemangioma is the most common noncancerous tumor of the liver. It is


believed to be a birth defect. Hepatic hemangiomas can occur at any time, but are
most common in people in their 30s - 50s. Women are affected more often than
men, and usually have bigger tumors than men. Babies may develop a type of
hepatic hemangioma called benign infantile hemangioendothelioma (also called
multinodular hepatic hemangiomatosis). This rare, noncancerous tumor has been
linked to high rates of heart failure and death in infants. Infants are usually
diagnosed by the time they are 6 months old.
272) Child with large periorbital hemangioma , if this hemangioma cause obstruction
to vision , when will be permenant decrease in visual acuity

a. After obstruction by one day

b. By 1 week

c. By 3 months

d. By 6 months

?????????

…………………………………………………………………………………………………………………………………………………

273) The symptoms of soft tissue sarcoma

a. Progressive enlarging mass ( on growing )

In their early stages, soft tissue sarcomas usually do not cause symptoms.
Because soft tissue is relatively elastic, tumors can grow rather large, pushing
aside normal tissue, before they are felt or cause any problems. The first
noticeable symptom is usually a painless lump or swelling. As the tumor grows, it
may cause other symptoms, such as pain or soreness, as it presses against nearby
nerves and muscles. If in the abdomen it can cause abdominal pains commonly
mistaken for menstrual cramps, indigestion, or cause constipation

…………………………………………………………………………………………………………………………………………………

274) 35 year female with bilateral breast pain , that decrease after menstruation , the
breast is nodular with prominent 3 cm mass subareolar , axillary lymph nodes are
not enlarged , what you will do

a. Mammography followed by US

b. See her next cycle

c. Fine needle biopsy followed by tissue studies

the correct answer is b


275) Pregnant with bleeding for 12 hours and tissue , the cervix is 1 cm

a. Complete abortion
. b. Incomplete abortion
. c. Molar pregnancy

the correct answer is b


276) 2 month infant with vomiting after each meal , he is in 50 centile , He passed
meconium early and stool , diagnosis is

a. Midgut volvulus

b. Meconium ileus (not passing early meconium and stool)

c. Hischsprung disease (not passing early meconium and stool)

I think this case is pyloric stenosis. If not mention choose a

…………………………………………………………………………………………………………………………………………………

277) Patient with dysphagia to solid and liquid , and regurg , by barium there is non
peristalsis dilatation of osophagus and air-fluid level and tapering end.diagnosis is

a. Osophageal spasm

b. Achalasia

c. Osophageal cancer

the correct answer is b

…………………………………………………………………………………………………………………………………………………

278) The most common cause of cough in adults is

a. Asthma
b. GERD
c. Postnasal drip

The correct answer c

The most common causes of acute cough is respiratory viral infection. The most
common cause of chronic cough are postnasal drip, asthma, and acid reflux from
the stomach. These three causes are responsible for up to 90 percent of all cases of
chronic cough.
279) Girl with amenorrhea for many months . BMI is 20 and is stable over last 5 years .
diagnosis

a.Eating disorder

b.Pituitary adenoma

…………………………………………………………………………………………………………………………………………………

280) Old female with itching of vulva , by examination there is pale and thin vagina ,
no discharge . what is management

a. Estrogen cream

b. Corticosteroid cream

c. Fluconazole

the correct answer is a

Some women will opt for prescription medications as they go through menopause.
The most common prescriptions include:
1 -Hormone replacement therapy or anti-depressants to minimize hot flashes.
2 -Fosamax or Actonel (non-hormonal medications) to reduce bone loss and reduce
the risk of fractures .
3 -Selective estrogen receptor modulators (SERMs), which mimic estrogen's
beneficial effects on bone density.
4-Vaginal estrogen, administered locally, to relieve vaginal dryness and discomfort
during intercourse

…………………………………………………………………………………………………………………………………………………

281) Patient with dysuria , frequency , urgency , but no flank pain , what is the
treatment

a. Ciprofloxacin po od for 3-5 days

b. Norfocin po od for 7 – 14 days

the correct answer is b


282) Patient with flank pain , fever , vomiting , treatment is

a. Hospitalization and intravenous antibiotics and fluid

This is most likely a case of pyelonephritis which need urgent hospitalization

…………………………………………………………………………………………………………………………………………………

283) Patient present with mid face pain , erethematous lesions and vesicles on
periorbital and forehead , the pain is at nose , nose is erythematous . what is
diagnosis

a. Roseola
. b. HSV
. c. Herpes zoster

the correct answer is c

…………………………………………………………………………………………………………………………………………………

284) Male with itching in groin erythematous lesions and some have clear centers ,
what is diagnosis :
a. Psoriasis
b. Tinea curis
c. Erythrasma

the correct answer is b

Patients with tinea cruris report pruritus and rash in the groin. A history of previous
episodes of a similar problem usually is elicited. Additional historical information in
patients with tinea cruris may include recently visiting a tropical climate, wearing
tight-fitting clothes (including bathing suits) for extended periods, sharing clothing
with others, participating in sports, or coexisting diabetes mellitus or obesity. Prison
inmates, members of the armed forces, members of athletic teams, and people
who wear tight clothing may be subject to independent or additional risk for
dermatophytosis.Large patches of erythema with central clearing are centered on
the inguinal creases and extend distally down the medial aspects of the thighs and
proximally to the lower abdomen and pubic area.-
285) Vasoconstrictive nasal drops complication

a. Rebound phenomenon

…………………………………………………………………………………………………………………………………………………

286) The useful excurcise for osteoarthritis in old age to maintain muscle and bone
Low resistance and high repetion weight training:

a. Conditioning and low repetion weight training

b. Walking and weight exercise

the correct answer is b

Exercise is one of the best treatments for osteoarthritis. The best exercises
for osteoarthritis suffers depend on what joints are affected. Swimming,
walking, and cycling are often the best exercises for people with osteoarthritis.
Try to get thirty minutes of exercise five times per week. The key is to start
slowly.

…………………………………………………………………………………………………………………………………………………

287) Unilateral worsening headach , nausea , excacerbeted by movement and


aggrevated by light in 17 old girl.

a. Migraine (Photophobia, vomiting)


b. Cluster

the correct answer is a


288) Diet supplement for osteoarthritis

a. Ginger

A large number of dietary supplements are promoted to patients with


osteoarthritis and as many as one third of those patients have used a supplement
to treat their condition. Glucosamine-containing supplements are among the most
commonly used products for osteoarthritis. Although the evidence is not entirely
consistent, most research suggests that glucosamine sulfate can improve symptoms
of pain related to osteoarthritis, as well as slow disease progression in patients with
osteoarthritis of the knee. Chondroitin sulfate also appears to reduce osteoarthritis
symptoms and is often combined with glucosamine, but there is no reliable
evidence that the combination is more effective than either agent alone. S-
adenosylmethionine may reduce pain but high costs and product quality issues limit
its use. Several other supplements are promoted for treating osteoarthritis, such as
methylsulfonylmethane, Harpagophytum procumbens (devil's claw), Curcuma longa
(turmeric), and Zingiber officinale (ginger), but there is insufficient reliable
evidence regarding long-term safety or effectiveness.

………………………………………………………………………………………………………………………………………………

289) Old male with abdominal pain , nausea , WBC 7. What is true about appendicitis
in elderly?

a. Ct not usefull for diagnosis.


b. WBC is often normal.
c. Rupture is common
d. If there is no fever the diagnosis of appendicitis is unlikely
e. Anemia is common

the correct answer is c

Appendicitis in elderly
Appendicitis is a less common cause of abdominal pain in elderly patients than in younger patients, but
the incidence among elderly patients appears to be rising. Only approximately 10% of cases of acute
appendicitis occur in patients older than 60 years, whereas one half of all deaths from appendicitis
occur in this age group. The rate of perforation in elderly patients is approximately 50%, 5 times higher
than in younger adults. This is largely because 75% of elderly patients wait more than 24 hours to seek
medical attention. The diagnosis can be difficult to make, since more than one half of patients in this
age group do not present with fever or leukocytosis. Further confusing the picture, approximately one
third do not localize pain to the right lower quadrant, and one fourth do not have appreciable right
lower quadrant tenderness. Only 20% of elderly patients present with anorexia, fever, right lower
quadrant pain, and leukocytosis. The initial diagnosis is incorrect in 40-50% of patients in this age
range . ( perforation is the most common complication of appendicitis )
290) Old patient with bilateral enlarged knee , no history of trauma , no tenderness ,
normal ESR and C-reactive proteins . the diagnosis is

a. Osteoarthritis
b. Gout
c. Infectous arthritis

the correct answer is a ( I'm not sure )

………………………………………………………………………………………………………………………………………………

291) Patient has decrease visual acuity bilateral , but more in rt side , visual field is not
affected , in fundus there is irregular pigmentations and early cataract formation .

a. Refer to ophthalmologist for laser therapy

b. Refer to ophthalmologist for cataract surgery ??????????

the correct answer is a

………………………………………………………………………………………………………………………………………………

292) What is the most common treatment for juvenile rheumatoid arthritis

a. Intraarticular injection of steroid

b. Oral steroid

c. Paracetamol

d. penicillamine

e. Asprin

the correct answer is e

the most common important for JRA is ( NSAID )

……………………………………………………………………………………………………………………………………………

293) Which of the following decrease mortality after MI

a. Metoprolol
b. Nitroglycerine
c. Thiazide
d. Morphine

the correct answer is a


only b-blocker ans ASA are mortality benefit for treatment of angina
294) The cardiac arrest in children is uncommon but if occur it will be due to

a. Primary

b. Respiratory arrest

c. hypovolemic shock

d. neurogenic shock

the correct answer is b

Progressive respiratory insufficiency accounts for 60% of all paediatric arrests.

………………………………………………………………………………………………………………………………………………

295) Old female with recurrent fracture , Vitamen D insufeciency and smoker . which
exogenous factor has the gretest exogenous side effect on osteoporosis.

a. Old age

b. Smoking

c. . Vit D insufeciency

d. Continue smoking

e. Recurrent fracture

the correct answer is b

………………………………………………………………………………………………………………………………………………

296) patient presented with sudden chest pain and dysnea , tactile vocal fremitus and
chest movemebt is decreased , by x-ray there is decreased pulmonary marking in
left side , diagnosis

a. atelectasis of left lung

b. spontaneous pneumothorax

c. pulmonary embolism

??????????
297) boy after running for hours , has pain in knee and mass on upper surface of tibia

a. Osgood scatter disease

b. Iliotibial band

the correct answer is a

………………………………………………………………………………………………………………………………………………

298) pancreatitis

a. Amylase is slowly rising but remain for days

b. Amylase is more specific but less sensitive than lipase

c. Ranson criteria has severity (predictive) in acute pancreatitis

d. Pain is increased by sitting and relieved by lying down

e. Contraceptive pills is associated

the correct answer is c

The Ranson and Glasgow scoring systems are based on such parameters and have
been shown to have an 80% sensitivity for predicting a severe attack, although only
after 48 hours following presentation.

risk mortality is 20% with 3-4 signs, 40% with 5-6 signs, 100% 7 signs.
299) Patient has fever , night sweating , bloody sputum , weight loss , ppd test was
positive . x-ray show infiltrate in apex of lung , ppd test is now reactionary ,
diagnosis

a. Activation of primary TB

b. sarcoidosis

c. Case control is

d. Backward study

the correct answer is a

The tuberculosis skin test (also known as the tuberculin test or PPD test) is a test
used to determine if someone has developed an immune response to the
bacterium that causes tuberculosis (TB).

……………………………………………………………………………………………………………………………………………………..

300) patient with DM presented with limited or decreased range of movement passive
and active of all directions of shoulder
a. frozen shoulder
b. impingment syndrome
c. osteoarthritis

the correct answer is a


……………………………………………………………………………………………………………………………………………………..

301) 48 years old with irregular menses presented with fatigue and no menstruation
for 3 months with increased pigmentation around the vaginal area with no other
symptoms. ur next step would be
a. reassure the patient
b. do a pregnancy test
c. do ultrasound

the correct answer b


302) 3 years old presented with shortness of breath and cough at night which resolved
by itself in 2 days. he has Hx of rash on his hands and allergic rhinitis. he most
likely had
a. croup
b. bronchial asthama
c. epilotitis

????????

Croup Epiglottitis

Onset Days Hours

Flu-like symptoms Yes No

Cough Sever Absent

Able to drink Yes No

Drooling saliva No Yes

Fever <38 >38

Stridor Harsh Soft

Voice Hoarse muffed

…………………………………………………………………………………………………………………………………………………

303) a man went on vacation. he noticed a white patch in his chest which became
more clear after getting a sun tan which was spread on his chest.ur Dx is
a . pytriasis versicolor
b. vitilligo
c. pytriasis roscea

the correct answer a


……………………………………………………………………………………………………………………………………………………..
304) a 4 years old presented with 2 day history of shortness of breath a seal like cough
with no sputum and mild fever. on examination he did not look I'll or in distress

a. acute epilossitis
b. croup
c. angioedema

The correct answer b


304)65 years old came with knee pain. and limited movement. on examination had
crepition on knee. dx
a. rhuematiod arthritis
b. osteoporosis
c. osteoarthritis

the correct answer c

……………………………………………………………………………………………………………………………………………………..
305) which vitamin is given to new born to stop bleeding
a. vit. A
b. vit. D
c. vit. K
d. vit E
e. vit C

the correct answer c

……………………………………………………………………………………………………………………………………………………..

306) a child came with congested throught and mild bulging of tympanic membrane ur
Dx was URTI. The
?????????

……………………………………………………………………………………………………………………………………………………..

307) a child presented with erythematous pharynx, with cervical lymph nodes and
rapid strplysin test negative and low grade fever with positive EBV. it next step
a. give antibiotics and anti pyretic
b. give anti pyretic and fluids
d. culture and sensitivity

the correct answer a


……………………………………………………………………………………………………………………………………………………..

308) child with low grade fever and congested throat, negative ASO and positive EBV.
he has
a. infectous mononucleosis
b. URTI

the correct answer is a


……………………………………………………………………………………………………………………………………………………..
309) the most common cause of intracerebral or intraparynchimal bleeding is.
a. Hypertensive angiopathy
b. annurysm
c. AV malformation

the correct answer is a


310) young adult Sickle cell patients are commonly affected with
a. dementia
b. multiple cerebral infarcts

the correct answer b

……………………………………………………………………………………………………………………………………………………..

311) 80 years old living in nursing home for the last 3 months. his wife died 6 months
ago and he had a cornary artery disease in the last month. he is now forgetful
especially of short term memory and decrease eye contact with and loss of
interest. dx
a. alzihiemer
b. depression
c. hypothyroidism

the correct answer a , I think Q is not complete

……………………………………………………………………………………………………………………………………………………..

312) thyroid cancer can be from


a. hypothyroidism
b. graves disease
c. toxic nodule

?????????

……………………………………………………………………………………………………………………………………………………..

313) a mother came with her son who is 7 years old very active never sitting in class
and with poor concentration. ur management would be.
a. olanzipine
b. amitilyne
c. aloxane

the correct answer is b ( I'm not sure )

this case is "attention –deficit hyperactivity disorder"


treatment:
1- Methylphenidate
2- Dextroamphetamine
3- Aderall
4- Atomoxetine
5- Pemoline
6- Antidepressant ( SSRI )
314) from the graph above which of the following statement is true
a. osteoprosis affecting majority of women above 80
b. 20% of women under 70 are affected

?????????
……………………………………………………………………………………………………………………………………………………..

315) a patient presented with progressive weakness on swollowibg with diplopia and
fatigability. the most likely underlying cause of her disease is.
a. antibody against acetylcholine receptors

……………………………………………………………………………………………………………………………………………………..
316) an 69 year old non diabetic. with mild hypertension and no hx of Coronary heart
desease. the best drug in treatment is.
a. thiazides
b. ACEI
c. ARB
d. CCB

the correct answer a


……………………………………………………………………………………………………………………………………………………..

317) which of the following anti hypertensive is contraindicated for an uncontrolled


diabetic patient
a. hydrochlorothiazide
b. Losartan
c. hydralszine
d. spironolactone

the correct answer is a


-side effect of thiazide : hyperglycemia
……………………………………………………………………………………………………………………………………………………..

318) a wound stays in it's primary inflammation untill


a. Escher formation
b. epitheliazation
c. after 24 hours
d. wound cleaning

the correct answer d


……………………………………………………………………………………………………………………………………………………..
319) 70 years old male patient with mild urinary dripping and hesitency ur Dx is mild
BPH. ur next step in management is
a. transurethral retrograde prostatectomy
b. start on medication
c. open prostatectomy

the correct answer b


320) most common cause of secondary HTN is
a. Renal disease - Renal parenchymal disease

……………………………………………………………………………………………………………………………………………………..

321) one of the following food is known to reduce cancer


a. fibers
……………………………………………………………………………………………………………………………………………………..

322) smoking is a definitive risk of


a. sq.c.ca of bladder
b. liver
c. breast

the correct answer is a

……………………………………………………………………………………………………………………………………………………..

323) exercise recommended for patients with CAD. is


a. isometric
b. isotonic
c. yoga

‫؟؟؟؟؟؟؟؟؟؟‬

……………………………………………………………………………………………………………………………………………………..

324)what is the definitive treatment of frostbites


a, Rewarming

……………………………………………………………………………………………………………………………………………………..
325) the most common cause of failure to thrive in pediatric is
a. malnutrition

……………………………………………………………………………………………………………………………………………………..

326) which of the following causes the highest maternal mortality in pregnancy
a. toxoplasma
b. hyperbilirubenia
c. ????

" The major causes of maternal death are bacterial infection, variants of
gestational hypertension including pre-eclampsia and HELLP syndrome,
obstetrical hemorrhage, ectopic pregnancy, puerperal sepsis (childbed fever),
327) Ttt of frostbite
a. Immersion in water 40-45 C
b. Debridement
c. Leave it at room tepmrature

The correct answer is a

……………………………………………………………………………………………………………………………………………………..
328) Case about a child both RBS, FBS are elevated so he has DM1…what's the type of HLA
a.DR3
b.DR4
c.DR5
d.DR6
e.DR7

the correct answer is a

Types of HLA :
DR2 : Good's pasture syndrome & multiple myeloma
DR3 : D.M , SLE , & grave's disease
DR27 : ankylosing spondylaitis & reiter's syndrome
B51 : behceat's disease
D11 : hashimot's disease

…….……………………………………………………………………………………………………………………………………………..
329) Definition of PPV ?

Predictive positive value ( PPV ) : proportion of people with a positive test who have s a
disease : a / (a+b)

……………………………………………………………………………………………………………………………………………………..
330) What's the most common case in PHC centers
a.UTI
b.HTN
c. Coryza

the correct answer is c


331) all of the following will decrease pt compliance except:
a. involve pt in the plan
b. make simplified regimen
c. give easy written instructions
d. make appointments flexible
e. warn the patient about the danger of missing a pill

the correct answer is e

……………………………………………………………………………………………………………………………………………………..
332) Case about old diabetic patient who still have hyperglycemia despite increase insulin
dose…the problem with insulin in obese patients is
a. Post receptor resistance
?????? Q NOT COMPLETE

…………………………………………………………………………………………………………………………………………………….

333) Case about a child with drooling, fever, barking cough in sitting position, dx:
a. Croup
b. Broncholities
c. Pneumonia

The correct answer is a

……………………………………………………………………………………………………………………………………………………..
334) Mother came to you after her son had hematoma under the nail dt injury:
a. Send home with a pad on the head
b. Send home with acetaminophen
c. Do wedge resection
d. Evacuate the hematoma

The correct answer is d

This case is ( subungual hematoma )


335) What vaccine u'll give to a SCD ( sickle cell disease ) child
a.HBV
b. H.influenza
c. pneumococcal
d. both A and B
e. all of the above

the correct answer is c

……………………………………………………………………………………………………………………………………………………..

336) a mother with HBsAg positive came with her child 6 yrs old who has HBsAg +ve what
will you give him:
a. oral polio, DTP, MMR

the correct answer is a


other choices all had HBV vaccine which u'll not give

……………………………………………………………………………………………………………………………………………………..

337) Patient wil LLQ pain, vomiting, fever, high WBC (17.000), tenderness and rebound
tenderness
a. Diverticulitis
b. Sigmoid volvulus
c. Appendicitis
d. Toxic enteritis

The correct answer is a

……………………………………………………………………………………………………………………………………………………..

338) What's the organism responsible for psuedomembranous colitis:


a. Pseudomonas
b. Colisteridum
c. E.coli
d. Enterococcus fecalis

The correct answer is b


339) Mother came with her child who had botillism, what you will advice her:
a. Never eat canned food again
b. Store canned food at home
c. Boil canned food for 40-50 min
d. Check expiry date of canned food

The correct answer is d

……………………………………………………………………………………………………………………………………………………..

340) Old pt presented with abdominal pain, back pain, pulsatile abdomen what's the step
to confirm dx: this is a case of aortic aneurysm
a. Abdominal US
b. Abdominal CT
c. Abdominal MRI

The correct answer is a b


-initial investigation US – CONFIRM by CT

……………………………………………………………………………………………………………………………………………………..

341) 18 months old came with bite by her brother, what you will do:
a. Give augminten
b. Give titunus toxoid
c. Suture

Q not complete

……………………………………………………………………………………………………………………………………………………..

342) 19 yrs old after bike accident, he cant bring the spoon infront of himself to eat, lesion
is in:
a. Temporal lobe
b. Cerebellum
c. Parietal lobe
d. Occipital lobe

The correct answer is b


343) How to dx DVT:
a. Contrast venography
b. Duplex US

The correct answer is b

……………………………………………………………………………………………………………………………………………………..

344) Pt came after RTA, GCS 14, near complete amputation of the arm, 1st step:
a. Secure air way
b. Tourniquet on the arm

The correct answer is a

……………………………………………………………………………………………………………………………………………………..

345) Pt with hx of prolonged heavey bleeding 2 hrs post partum, you will give:
a. Ringers lactate
b. NS
c. NS+ packed erythrocytes

The correct answer is c ( Q NOT COMPLETE )

……………………………………………………………………………………………………………………………………………………..
346) What is special about placenta abruption:
a. Abnormal uterine contractions
b. PV bleeding
c. Fetal distress

The correct answer is b , PA : dark , painful vaginal bleeding and complication is fetal
hypoxia

……………………………………………………………………………………………………………………………………………………..

347) Pt 34 wks, hx of PV bleeding for many hrs, dark blood, abdominal tenderness, FHR
120, uterine contractions every 3 min, Dx:
a. Abruption placenta
348) One of the steps in managing epistaxis:
a. Packing the nose
b. Press the fleshy parts of nostrils
c. Put patient of lateral lying position

The correct answer is b

……………………………………………………………………………………………………………………………………………………..

349) Pt with hx of diarrhea, abdominal pain, agitation, headache, dizziness, weakness,


pulstile thyroid, unsteady gate. Examination was normal. Dx:
a. Hypochondriasis
b. Somatization disorder
c. Thyroid Ca
d. Anxiety

The correct answer is b

……………………………………………………………………………………………………………………………………………………..

350) Child with headache that increase by changing head postion, unilateral, photophobia
a. Sinus headache
b. Migraine
c. Cluster headache
d. Tension headache

The correct answer is b

……………………………………………………………………………………………………………………………………………………..

351) Ttt of isolated fracture of femur (repeated) ???

………………………………………………………………………………………………………………………………………………………

352) Younge pt with pain in LL after running 3 kgs, more at night, swelling, XR was NL
a. Stress fracture
353) HTN pt, with decrease vision, fundal exam showed increase cupping of optic disc dx:
a. Open angle glaucoma
b. Closed angle glaucoma
c. Cataract
d. HTN changes

The correct answer is a ( I'm not sure )

……………………………………………………………………………………………………………………………………………………..

354) Pt with unilateral red eye, tearing, clear tears no swelling or discharge:
a. Give topical antihistamin
b. Give topical AB
c. Give systemic AB
d. Reassurance

The correct answer is d

……………………………………………………………………………………………………………………………………………………..

355) Business man went to Pakistan, came with bloody diarrhea, stool examination showed
trophozoite with RBC inclusion, Dx:
a. Amebic desyntry (entamoeba histlolytica )

……………………………………………………………………………………………………………………………………………………..

356) 5 yrs old child with abdominal pain after 2 wks of URTI, HB 8, retics 12% WBC NL
peripheral blood smear showed target cells, RBC inclusions dx:
a. SCA (the only hemolytic anemia in the answers)
?????

……………………………………………………………………………………………………………………………………………………..

357) Child had hb electrophoresis showed hb 2%


a. HBA 40 HBS 35 HBF 5 HBA2
b. Sickle cell trait
c. Thalacemia major
d. Minor
?????
358) Younge pt with hx of cough, chest pain, fever CXR showed RT lower lobe infiltrate:
a. Amoxicillin
b. Ceferuxim
c. Emipenim
d. Ciprofloxacin
??????
The correct answer is d (I'm not sure )

……………………………………………………………………………………………………………………………………………………..

359) Best thing to reduce mortality rate in COPD:


a. Home O2 therapy
b. Enalipril
c. Stop smoking

The correct answer is c

……………………………………………………………………………………………………………………………………………………..

360) Drug that will delay need of surgery in AR:


a. digoxin
b. verapamil
c. nefidipin
d. enalpril

the correct answer is c

……………………………………………………………………………………………………………………………………………………..
361) Child with skin rash, pericarditis, arthritis dx:
Kawasaki

……………………………………………………………………………………………………………………………………………………..

362) Pic of skin with purple flat topped polygonal papules, dx:
a. Lichen plannus

……………………………………………………………………………………………………………………………………………………..

363) Child with atopic dermatitis, what you will give other than cortisone
a. There were many drugs, steroids and AB and only one strange name maybe
moisturizer. That's the answer
364) A man had increase shoe size and jaw, the responsible is:
a. ACTH
b. Somato??
c. TSH
d. Cortisone

The correct answer is b

……………………………………………………………………………………………………………………………………………………..

365) The best advice to patient travelling is:


a. Boiled water
b. Ice
c. Water
d. Salad and under cooked sea shells

The correct answer is a

……………………………………………………………………………………………………………………………………………………..

366) Seldinfil is contraindicated with:


a. Nitrate
b. Methyldopa
c. Gabapentine

The correct answer is a

……………………………………………………………………………………………………………………………………………………..

367) Pt with TB, had ocular toxicity symptoms, the drug responsible is:
a. INH
b. Ethambutol
c. Rifampicin
d. Streptomycin

The correct answer is b

INH : peripheral neuritis and hepatitis . so add ( B6 )


Ethambutol : optic neuritis
Rifampicine : body fluid organe
368) Another pt treated for TB started to develop numbness, the vit deficient is:
a. Thiamin
b. Niacin
c. Pyridoxine
d. Vit C

THE correct answer is c

……………………………………………………………………………………………………………………………………………………..

369) Pt with bilateral breast disease, dx:


a. Paget
b. Papilloma
c. Meduallary
d. Lobular

……………………………………………………………………………………………………………………………………………………..

370) Child fell on her elbow and had abrasion, now swelling is more, tenderness, redness,
swelling is demarcated (they gave dimensions) child has fever. Dx:
a. Gonoccal arthritis
b. Synovitis
c. Cellulitis of elbow

The correct answer is c

……………………………………………………………………………………………………………………………………………………..

371) About burn


a. You will give 1\2 fluid in the 1st 8 hrs
b. 1\4 in the 1st 8 hrs

The correct answer is a

Parkland formula : fluid in first 24 hours = 4 * wt. in kg * % BSA


50% in first 8 hours and another in next 16 hours
372) You r supposed to keep a child NPO he's 25 kgs, how much you will give:
a. 1300
b. 1400
c. 1500
d. 1600

The correct answer is d

For the first 10 kg of weight, a child needs 100 mL per kg of weight.


For the next 10 kg of weight (11-20 kg), a child only needs 50 mL per kg of weight.
for anything over 20 kg (21 kg of weight and higher), the child only needs 20 mL per kg
of weight.
Take a 35 kg child, for example. He needs 1000 mL for his first 10 kg of weight (10 kg x
100 mL), 500 mL for his second 10 kg of weight (10 kg x 50 mL), and 20 mL/kg for any
weight above 20 kg (15 kg x 20 mL). A 35 kg child, therefore, needs approximately
1800mL of water or free liquids.
……………………………………………………………………………………………………………………………………………………..

373) Young patient with pharyngitis, inflammation of oral mucosa and lips that has whitish
cover and erythmatous base, febrile, splenomegaly. Dx: (this is infectious mono)
a. Scarlet fever
b. EBV
c. HZV

The correct answer is b

……………………………………………………………………………………………………………………………………………………..

374) Female with greenish vaginal discharge, red cervix. Dx:


Trchimoniosis

……………………………………………………………………………………………………………………………………………………..

375) Another female with malodorous discharge and pain maybe. Dx:
Bacterial vaginosis

……………………………………………………………………………………………………………………………………………………..
376) The best way to reduce the weight in children is:
a. stop fat intake
b. Decrease calories intake
c. Drink a lot of water

The correct answer is b


377) You have to advice a teenager that mainly eats fast food to take:
a. Folic acid and Ca
b. Vit C and Ca
c. Ca alone

The correct answer is c ( I'm not sure )

……………………………………………………………………………………………………………………………………………………..

378) Patient has symptoms of infection, desquamation of hands and feet, BP 170\110 dx:
a. Syphilis
b. Toxic shock syndrome
c. Scarlet fever

The correct answer is b

Toxic shock syndrome :


Caused by S.aureus , often with 5 days of onset of menestral period in women who
have used tampons. Feature: abrupt fever ( 39 c or more ), vomiting, diffuse macular
erythematous rash, desquamation especially in palms and soles, nonpurulent
conjunctivitis. Diagnosis : blood culture are –ve .so, diagnosis by clinical. Treatment :
1st step rehydration and antibiotic

……………………………………………………………………………………………………………………………………………………..
379) Snellin chart, if patient can read up to line 3, how much is his vision loss :

Q not complete:
but this pt. see letters at 20 feet , where normal person see it at 70 feet.
380) I cant remember the case but gram +ve cocci were isolated, dx is sterpt bcz it was the
only gm +ve cocci.

……………………………………………………………………………………………………………………………………………………..

381) Diabetic mother asking about risk of diabetes related congenital defect. It will be in:
a. 1st trimester
b. 2nd trimester
c. 3rd

The correct answer is a (I'm not sure )

……………………………………………………………………………………………………………………………………………………..

382) If diabetic mother blood sugar is always high despite of insulin, neonate complication
will mostly be:
a. Maternal hyperglycemia
b. Maternal hypoglycemia
c. Neonatal hypoglycemia
d. Neonatal hyperglycemia

The correct answer is c

……………………………………………………………………………………………………………………………………………………..

383) Obssive neurosis patients will have:


a. Major depression
b. Lake of insight
c. Schizophrenia

The correct answer is a

……………………………………………………………………………………………………………………………………………………..

384) Before giving bipolar patient lithium you will do all of the following except:
a. TFT
b. LFT
c. RFT
d. Pregnancy test

The correct answer is b


385) Patient came with PV small bleeding (she fell down and felt decrease in fetal
movement but on examination baby was ok) all her 3 previous pregnancies were
normal, you will do:
a. Immediate CS
b. Careful observation of the bleeding
c. Medication (strange name)
d. Mg sulphate

Q not complete , depend on date of gestation.

……………………………………………………………………………………………………………………………………………………..
386) Pt came to you missing her period for 7 wks, she had minimal bleeding and abdominal
pain, +ve home pregnancy test, 1st thing to order is:
a. BHCG
b. US
c. Drugs maybe

The correct answer is a

……………………………………………………………………………………………………………………………………………………..

387) Patient came to you and you suspect pre eclampisa, which of the following will make
it most likely:
a. Elevated blood pressure
b. Decrease fetal movement
c. ??

The correct answer is a

……………………………………………………………………………………………………………………………………………………..

388) Old patient male, hematuria, passing red clots and RT testicular pain:
a. Testicular Ca
b. RCC ( renal cell carcinoma )
c. Cystitis

The correct answer is b


389) to get more information from the pt
a. open end question

( avoid leading question to get more information )

……………………………………………………………………………………………………………………………………………………..
390) gingivits most likely cause
a.HSV

( q not complete but : The most common cause of gingivitis is poor oral hygiene that
encourages plaque to form.)

……………………………………………………………………………………………………………………………………………………..
391) watery discharge from eye , conjunctivitis treated by
a. topical corticosteroid

(Its symptoms include watery discharge and variable itch. The infection usually
begins with one eye, but may spread easily to the other
Ttt: cold compresses ] and artificial tears)

……………………………………………………………………………………………………………………………………………………..
392) blow out fracture eyelid swelling , redness other syptoms
a. present air fluid level
b. enopthalmos

The correct answer is b


its enophaloms , other name of blow out orbital floor fracture

……………………………………………………………………………………………………………………………………………………..
393) drinking of dirty water causes
a. heptitis A
b. B
c. C
d. D

The correct answer is a

……………………………………………………………………………………………………………………………………………………..

394) giemsa stained blood film


a. malaria
395) regarding peritonitis
a.Complicated appendectomy the cause is anerobe organism
b. rigidity and the cause is paralytic ileus
c. can be caused by chemical erosions.

The correct answer is c

……………………………………………………………………………………………………………………………………………………..
396) fraucture of humerus associated with
a. radial N injury
b. median nerve injury

the correct answer is a ( radial groove )

……………………………………………………………………………………………………………………………………………………..

397) there is case i cant remember they asked about diagnosis of acute lymphocytic
leukemia ALL

The total number of white blood cells may be decreased, normal, or increased, but
the number of red blood cells and platelets is almost always decreased. In addition,
very immature white blood cells (blasts) are present in blood samples examined
under a microscope.
A bone marrow biopsyis almost always done to confirm the diagnosis and to
distinguish ALL from other types of leukemia

……………………………………………………………………………………………………………………………………………………..
398) diagnosis of hemochromatosis
a. serum ferritin?!

Hemochromatosis is suggested by a persistently elevated transferrin saturation in


the absence of other causes of iron overload. It is the initial test of choice .
Ferritin concentration can be high in other conditions, such as infections,
inflammations, and liver disease .
Ferritin levels are less sensitive than transferrin saturation in screening tests for
hemochromatosis .
399) kawasaki disease associated with
a.strawberry tongue

 Kawasaki disease
Multisystem acute vasculitis that primary affected young children. Fever plus
four or more of the following criteria for diagnosis:
1-fever > 40 C for at least five days
2-bilateral, nonexudative, painless conjunctivitis
3-polymorphous rash ( primarily truncal )
4-cervical lymphadenopathy ( often painful and unilateral )
5- diffuse mucous membrane erythema ( strawberry tongue ) , dry red
6- erythema of palm and sole
7- other maindfestation : gallbladder hydrops, hepatitis, arthritis

Untreated Kawasaki diease can lead to coronary aneurysms and even MI


Treatment :
1-high dos ASA ( for fever and inflammation) & IVIG ( to prevent aneurrysmal )
2-referral to pediatric cardiologist

……………………………………………………………………………………………………………………………………………………..
400) most common tumor in children
a.ALL
b. rabdomyosarcoma
c. wilm's tumor

the correct answer is a

-ALL : most common childhood tumor


-rabdomyosarcoma : most common soft tissue tumor
-wilm's tumor: most common intra- abdominal childhood tuomr

……………………………………………………………………………………………………………………………………………………..
401) Most common intra- abdominal tumor in children:
a.wilm's tumor
b. lymphoma

the correct answer is a


the most common intra-abdominal tumor in children are neruoblastoma and
wilm's tumor.
402) pt c/o of hypopigmentful skin , nerve thicking diagnosis
a.leprosy

Leprosy : chronic granulomatous infection caused by Mycobacterium leprae, acid-fast


bacillus preferentially affecting cooler regions of body ( e.g skin , mucousmembrane,
peripheral nerve) . diagnosis by skin lesion ( hypopigmentation or hyperpigmentation
macule ) and enlarge nerves eith sensory loss.

……………………………………………………………………………………………………………………………………………………..
403) regrading COPD to reduce complication we should give
a. theophyline
b. pneumococcal vaccine
c. smoking cessation.

The correcr answer is c

……………………………………………………………………………………………………………………………………………………..

404) pseud-gout its


a. CACO3
b. CACL3

The correct answer is a


Gout : deposition of ( MSUM ) monosodium urate monohydrate –ve of birefringent
, needle shape,
Psudogout : deposition of ( cppd ) calcium pyrophosphates dehydrate crystal , +ve
birefringent , rhomboid shape, ( CACO3)

.
405) man with history of alcohol assocation with
a. high MCV
b. folic acid deficiency
c. B12 deficieny
e. hepatitis

The correct answer is b

……………………………………………………………………………………………………………………………………………………..
406) neanate 9 days on breast feed develop jandice
a. breast feed jaundice
b. pathological jaundice
c. physiological jaundice

the correct answer is a

1-pathological jaundice : conjugated ( direct ) bilirubin, start in 1st 24 hours, bilirubine >
15 mg /dl , persistent 1 week in term infant and 2 weeks in preterm infant

2-physilogical jaundice: unconjugated ( indirect ) bilirubine , start in 2- 3 days, bilirubine


< 15 mg/dl , persistent 1 week in term infant and 2 weeks in preterm infant.

3-breast feeding jaundice : unconjucated ( indirect ) bilirubine , start in 7 days of life ,


persistent to one or more months.

……………………………………………………………………………………………………………………………………………………..
407) lady c/o headche bandlike pain
a. tension headache

……………………………………………………………………………………………………………………………………………………..
408) regarding breast screening
a. self breast examination early detection of tumor
b. mammogram not advise before 35 y
??????

……………………………………………………………………………………………………………………………………………………..
409) 19 yrs old c/o abdo pain within menestration for last 6 years diagnosis
a. primary dysmenorrhea
b. secondary dysmenorrheal

The correct answer is a


Primary dysmenorrhea :
Onset within 6 months after menarche . Lower abdominal/pelvic pain begins with
onset of menses and lasts 8-72 hours
410) bilateral breast mass diagnosis
a. ductal carcinoma
b. pagets disease

it could be Intraductal papilloma ??????

……………………………………………………………………………………………………………………………………………………..
411) beriberi cause of deficiency
a. VIT B1
b. VIT B2
c. VIT B3

The correct answer is a


……………………………………………………………………………………………………………………………………………………..
412) chronic uses of estrogen association ?????

……………………………………………………………………………………………………………………………………………………..
413) Asystole
a. adrenalin
b. atropine

The correct answer is


asystol has only 3 durgs epinephrine-vasopressine-atropin

……………………………………………………………………………………………………………………………………………………..
414) pt has diarrhea , dermatitis and dementia diagnosis
a. pellagra

……………………………………………………………………………………………………………………………………………………..
415) regarding injectable progestron
a . an cause skin prob ?!!

Injected progesterone ( medroxy-progestrone ) : advantages : light or no periods,


safe with breastfeeding, IM injection every 3 months. Disadvantages: irregular
bleeding, wt. gain, decrease bone mineral density ( reversible ) , delay fertility
discontinue .

……………………………………………………………………………………………………………………………………………………..
416) sencodary prevention is
a. coronary bypass gaft

Secondary prevention generally consists of the identification and interdiction of


diseases that are present in the body, but that have not progressed to the point of
causing signs, symptoms, and dys-function
417) 22 yrs old c/o insomina/ sleep disturb treatment
a. SSRI

Short history ,
But the initial management of insomnia : good sleep hygiene .

……………………………………………………………………………………………………………………………………………………..
418) celluitis in children most commin causes
a. group A steptoccus

In children, the most common cause of cellulitis is S aureus or group A


streptococcus.

……………………………………………………………………………………………………………………………………………………..
419) antideprssants associated with hypertensive crisis treatment
a. SSRI
b. MOAIs
c. TCAs

the correct answer is b


esp hydrazines ( phenelzine )

……………………………………………………………………………………………………………………………………………………..

420) pt his MBI = 24 kg he is


a. noramal weight
b. over weight
c. morbid weight
d. mild weight

The correct answer is a

……………………………………………………………………………………………………………………………………………………..
421) baby c/o fever , chills , rigors and head rigidity +ve kurnings sign rash on his
lower limb diagnosis :
- meningoccal meninigits
???????

……………………………………………………………………………………………………………………………………………………..
422) 48 yrs pt with abdo pain , neusea, vominting tenderness in right hypochondrial
- acute cholecysitis
Fever ?!
423) 29 pt c/o dysurea his microscopic showed G -ve organism is
a. legonealla
b. E. coli

the correct answer is b

……………………………………………………………………………………………………………………………………………………..
424) 30 yrs pt c/o feeling heaviness in the lower abdomen having pulge papable at
the top scrotum that was reducible and icreasing in valsalva maneuver diagnosis
a. hydrocele
b. variocele
c. indirect inguinal hernia
d. direct inginal hernia

The correct answer is c

……………………………………………………………………………………………………………………………………………………..
425) anticoagulation prescrib for
a. one month
b. 6 months
c. 6 weeks
d. one year

The correct amswer is b

……………………………………………………………………………………………………………………………………………………..
426) cushing syndrome best single test to confirm
a. palsma cortison
b. ATCH
c. Dexamethasone Suppression Test

The correct answer is c

……………………………………………………………………………………………………………………………………………………..

427) 23 yrs old history of URTI then he developed ecchomysis best treated
a. local AB
b. local antiviral
c. steroid

The correct answer is c


428) chronic psycotic disorder manged by
a. haloperidol

……………………………………………………………………………………………………………………………………………………..
429) 29 yrs old lady B-HSG 160 c/o vomiting , abdomenal pain which is more
accurate to diagnosis
a. BHCG serial
b. pelvic US
c. laprascopy

Q not complete, but may be this case is molar and chooce is a .

……………………………………………………………………………………………………………………………………………………..
430) 70 yrs old man c/o fever , vesicular rash over forehead mangement
a. IV AB
b. IV antiviral
c. Acyclovir

the correct answer is c (i"m npt sure )

……………………………………………………………………………………………………………………………………………………..
431) celiac disease involves :
a. proximal part of small intestin
b. distal part of small intestin
c. proximal part of large intesin
d. distal part of large intensin

The correct answer is a

……………………………………………………………………………………………………………………………………………………..
432) 6 yes old pt cyanosis past history of similar attack 6 month ago u will do for
him
a. CxR
b. PFT
c. secure airway
d. CBC

Q is not clear or not complete , if pt. came with life threatening cyanosis don't waste
time with investigation and start with ABC ( scure airway ) , but if pt. came with past
history cyanosis choose b .
433) side affect of diazepam
Sedation , depence, respiratory supression , anterograde amnesia , confusion
(especially pronounced in higher doses) and sedation

……………………………………………………………………………………………………………………………………………………..

434) endemic means:

Endemic :is the constant presence of a disease or infectious agent in a certain


geographic area or population group. ( usually rate of disease )

Epidemic : is the rapid spread of a disease in a specific area or among a certain


population group. ( excessive rate of disease )

Pandemic : is a worldwide epidemic; an epidemic occurring over a wide


geographic area and affecting a large number of people.

……………………………………………………………………………………………………………………………………………………..

435) Epidemic curve :

a graph in which the number of new cases of a disease is plotted against an interval
of time to describe a specific epidemic or outbreak .
436) Pt with hodgkin's lymphoma , and red strunberg cell in pathology and there is
esinophil lymphocyte in blood so pathological classification is:

a. Mixed-cellularity subtype
b. nodular sclerosis subtype of Hodgkin's lymphoma

the correct answer is b

Classical Hodgkin's lymphoma can be subclassified into 4 pathologic subtypes


based upon Reed-Sternberg cell morphology and the composition of the reactive
cell infiltrate seen in the lymph node biopsy specimen (the cell composition
around the Reed-Sternberg cell(s))

Name descreption
Nodular sclerosing CHL Is the most common subtype and is composed of large tumor nodules showing scattered
lacunar classical RS cells set in a background of reactive lymphocytes, eosinophils and
plasma cells with varying degrees of collagen fibrosis/sclerosis.

Mixed-cellularity subtypeIs a common subtype and is composed of numerous classic RS cells admixed with
numerous inflammatory cells including lymphocytes, histiocytes, eosinophils, and plasma
cells. without sclerosis. This type is most often associated with EBV infection and may be
confused with the early, so-called 'cellular' phase of nodular sclerosing CHL
Lymphocyte-rich or Is a rare subtype, show many features which may cause diagnostic confusion with
Lymphocytic predominance nodular lymphocyte predominant B-cell Non-Hodgkin's Lymphoma (B-NHL). This form
also has the most favorable prognosis
Lymphocyte depleted Is a rare subtype, composed of large numbers of often pleomorphic RS cells with only
few reactive lymphocytes which may easily be confused with diffuse large cell
lymphoma. Many cases previously classified within this category would now be
reclassified under anaplastic large cell lymphoma
437) 62 y male with DVT and IVC obstruction due to thrombosis so most like dd is
a. neohrotic syndrome
b. SLE
C. Chirstm disease
D. ?!

……………………………………………………………………………………………………………………………………………………..

438) Pt with bdominal pain heamatutea , HTN, and have abnormalty in chromosm
16 , diagnosis is
a. POLY CYCTIC KIDNEY

……………………………………………………………………………………………………………………………………………………..

439) 17 year pt with dyspnea Po2 , PCO2 ,Xray normal PH increase so dd is:

a. acute attack of asthma


b. P E
c. pneumonia
d. pnemothrax

the correct answer is a

……………………………………………………………………………………………………………………………………………………..

440) A long scenario about patient with polydipsia ad polyuria. Serum osmolrity
high . desmoprsin inductin no change urine omolarity and plasma osmolrity so
dd is
a. nphrognic type
b. central tupe

The correct answer is a

……………………………………………………………………………………………………………………………………………………..

441) 50 year old Man presented to ER with sudden headach, blurred of vision and
eye pain. The diagnosis is:
a. Acute glaucoma
b. Acute conjunctivitis
c. Corneal ulcer
d. -----

The correct answer is a


442) RTA with hip dislocation and shock so causes of shock is
a. blood lose
b. urtheral injery
c. nurogenic

The correct answer is a

……………………………………………………………………………………………………………………………………………………..

443) most common causes of hand infection


a. truma
b. imunocrombromise
c. ………

the correct answer is a

……………………………………………………………………………………………………………………………………………………..

444) ttt of cholestatoma is


a. antibiotic
b. steroid
c. surgery
d. Grommet tube

the correct answer is c

-Cholesteatoma is a destructive and expanding growth consisting of keratinizing


squamous epithelium in the middle ear and/or mastoid . treatment : Surgery is
performed to remove the sac of squamous debris and a mastoidectomy is
performed.
445) twins one male and other female . his father notice that femle become
puberty before male so what you say to father

a. female enter puberty 1-2 year before male


b. female enter puberty 2-3 year before male
c. female enter puberty at the same age male

the correct answer is b

……………………………………………………………………………………………………………………………………………………..

446) in devolping countery to prevent dental carise , it add to water


a. florid
b. zink
c. copper
d. iodide

the correct answer is a

……………………………………………………………………………………………………………………………………………………..

447) 12 y.o boy c/o abdominal pain after playing football,he denied any h/o trauma
,the pain is in the Lt paraumbilical region what inx you want to do:
a. CXR
b. ultrasound kidney

The correct answer is b , I'm not sure because MCQs are not compete.

……………………………………………………………………………………………………………………………………………………..

448) pt child with back pain that wake pt from sleep


So diagnosis
a. lumber kyphosis
b. osteoarthritis
c. RA
d. Scoliosis

the correct answer is d


449) child with papule vesical on oropharnx and rash in palm and hand so dd:

a. CMV
b. EBV
c. MEASLS
d. ROBELLA

???? Coxsackie A virus. Vesical in buccal mucusa is measls.

……………………………………………………………………………………………………………………………………………………..

450) chid with dental caries and history of bottle feading


So dd
a. nurse milk caris

……………………………………………………………………………………………………………………………………………………..

451) Generalize antxity disorder best ttt: -19


a. Ssri
b. tricyclic a d
d. MAOI

The correct answer is a

……………………………………………………………………………………………………………………………………………………..

452) Major depression management:


a. Intial MONOTHERABY even sever sever deprisson
b. Ttt should be change if no response during 2wk (AT LEAST 6 . WEEKS )

????

……………………………………………………………………………………………………………………………………………………..

453) psychotherapy, medication, and electroconvulsive therap


Psychiatric pt with un compliance of drugs ttt:

a. depro halopredol injection


b. oral colonazepam

the correct answer is a


454) mild diarrhea mangement :
a. oral antibiotis
b. IVF
c. ORS

the correct answer is c

……………………………………………………………………………………………………………………………………………………..

455) salpingitis and pid on penicillin but not improve the most likely organism is :
a. chlamydia
b. nessiria
c. SYPHLIS
d. HSV

the correct answer b

……………………………………………………………………………………………………………………………………………………..
456) Used for treatment of pseudomembranous colitis:

PO metronidazole is the empiric treatment of choice (500 mg TID for 10-14


days). For severe infections, oral Vancomycin is the treatment of choice (125 mg
QID 10-14 days).

……………………………………………………………………………………………………………………………………………………..
457) Lady with of right hypochondrial pain, fever and slight jaundice. What is your
diagnosis?

a. Acute cholecystitis.

Q not complete.

Most common presenting symptom of acute cholecystitis is upper abdominal


pain often radiating to right scapula. Nausea and vomiting generally present.
Patients may report fever.
458) Deep jaundice wit palpable gallbladder ?

a. Cancer head of pancreas. .i

Q not complete

Pancreatic cancer presents with abdominal pain radiating toward the back, as
well as with
jaundice, loss of appetite, nausea, vomiting, weight loss, weakness, fatigue, and
indigestion.
Examination: may reveal a palpable, nontender gallbladder (Courvoisier’s sign)
or migratory thrombophlebitis (Trousseau’s sign). Diagnose by CT. treatment
usually palliative (due to metastasis at Dx).
……………………………………………………………………………………………………………………………………………………..

459) Patient with perforated gallbladder underwent cholecystectomy. Return to


you with fever. On abdominal X-ray, there is elevation of right hemidaiaphragm.
What is the possible diagnosis?

a. Subphrenic abcess

……………………………………………………………………………………………………………………………………………………..

460) Patient with acute perianal pain since 2 days with black mass )064
2*3 pain 4increase with defecation Rx:
a. Evacuation under local anesthesia

Q not complete

……………………………………………………………………………………………………………………………………………………..

461) Rhumatoid arthritis distal interphalengial nodules


a. Hebrerden's nodes

Heberden's nodes are at the DIP while Bouchard's nodes are at PIP.
462) Elderly patient with RLQ fullness, weight loss, changed bowel habit, anemic
and pale. What is the investigation of choice?

a. Colonoscopy.

This is now the investigation of choice if colorectal cancer is suspected provided the
patient is fit enough to undergo the bowel preparation. It has the advantage of not
only picking up a primary cancer but also having the ability to detect synchronous
polyps or even multiple carcinomas, which occur in 5% of cases.

……………………………………………………………………………………………………………………………………………………..
463) Investigation of choice in Iron deficiency anemia?
a. serum iron
b. ferritin
c. TIBC

the correct answer is b

Serum ferritin is the most sensitive lab test for iron deficiency anemia.

……………………………………………………………………………………………………………………………………………………..

464) In patients with hypertension and diabetes, which antihypertensive agent you
want to add first?
a. β-blockers
b. ACE inhibitor
c. α-blocker
d. Calcium channel blocker

the correct answer is b

Diuretics (inexpensive and particularly effective in African-Americans) and β-


blockers (beneficial for patients with CAD) have been shown to reduce mortality
in uncomplicated hypertension. They are first-line agents unless a comorbid
condition requires another medication. (see table)

Population Treatment
Diabetes with proteinuria ACEIs.
CHF β-blockers, ACEIs, diuretics (including
spironolactone).
Isolated systolic Diuretics preferred; long-acting dihydropyridine
hypertension calcium channel blockers.
MI β-blockers without intrinsic sympathomimetic
activity, ACEIs.
Osteoporosis Thiazide diuretics.
BPH α-antagonists.
465) Patient presented with retinal artery occlusion, which of the following is
wrong:
a. painful loss of vision
b. painless loss of vision

the correct answer is a

The most common presenting complaint of retinal artery occlusion is an acute


persistent painless loss of vision.

……………………………………………………………………………………………………………………………………………………..
466) ECG finding of acute pericarditis?
a. ST segment elevation in all leads

Classic ECG findings in pericarditis: Low-voltage, diffuse ST-segment elevation.

……………………………………………………………………………………………………………………………………………………..
467) Epidemiological study want to see the effect of smoking in the bronchogenic
carcinoma, they saw that is 90% of smokers has bronchogenic carcinoma.
30% of non–smokers has the disease, the specificity of the disease as a risk factor
is:
a. 70 %
b. 30 %
c. 90 %

MCQs not complete

Sensitivity: The probability that a diseased patient will have a positive test result.
Specificity: The probability that a nondiseased person will have a negative test result.

Disease No Disease
Present
Positive test a b
Negative c d
test

Sensitivity = a / (a + c) Specificity = d / (b + d)

Note:
- A sensitive test is good for ruling out a disease.
- High sensitivity = good screening test ( false negatives).
- High specificity = good for ruling in a disease (good confirmatory test).
468) Baby present with pain in the ear, by examination there is piece of a glass
deep in his ear canal, the mother mention a history of a broken glass in the
kitchen but she clean that completely. We treat that by:
a. By applying a stream of solution to syringing the ear.
b. Remove it by forceps.
c. Refer her to otolaryngology

* Foreign bodies in the ear canal:


Techniques appropriate for the removal of ear foreign bodies include mechanical
extraction, irrigation, and suction. Irrigation is contraindicated for organic matter
that may swell and enlarge within the auditory canal (e.g seeds). Insects, organic
matter, and objects with the potential to become friable and break into smaller
evasive pieces are often better extracted with suction than with forceps. Live
insects in the ear canal should be immobilized before removal is attempted.

Consultations: Consult an ENT specialist if the object cannot be removed or if tympanic


membrane perforation is suspected.

……………………………………………………………………………………………………………………………………………………..
469) Treatment of open tibial fracture:
a. cephazolin
b. cephazolin+gentamycin
c. gentamicin
d. cephazolin+gentamicin+metronidazole

treatment of open fracture

1- First-generation cephalosporins (Gram-positive coverage) such as cephalexin (1-2


g q6-8h) suffice for Gustilo type I open fractures.
2- An aminoglycoside (Gram-negative coverage) such as gentamycin (120 mg q12h;
240 mg/d) is added for types II and III injuries.
3- Additionally, metronidazole (500 mg q12h) or penicillin (1.2 g q6h) can be added
for coverage against anaerobes.
4- Tetanus prophylaxis should be instituted.
5- Antibiotics generally are continued for 72 hours following wound closure.

Open fracture classification


Gustilo classification
I Low energy, wound less than 1 cm
II Wound greater than 1 cm with moderate soft tissue damage
High energy wound greater than 1 cm with extensive soft tissue damage
IIIa Adequate soft tissue cover
III
IIIb Inadequate soft tissue cover
IIIc Associated with arterial injury
470) Which of the following found to reduce the risk of postherpetic neuralgia:
a) corticosteroids only
b) corticosteroids + valacyclovir
c) valacyclovir only

The correct answer is c

Patients with herpes zoster should receive treatment to control acute symptoms
and prevent complications. Patients over the age of 50, irrespective of other risk
factors, are at much greater risk of developing postherpetic neuralgia and should
be offered treatment. By inhibiting replication of varicella zoster virus, the antiviral
agents acyclovir, famciclovir, and valaciclovir attenuate the severity of zoster—
specifically, the duration of viral shedding is decreased, rash healing is hastened,
and the severity and duration of acute pain are reduced. Attenuation of the severity
of the acute infection and the neural damage it causes should reduce the likelihood
of postherpetic neuralgia.

……………………………………………………………………………………………………………………………………………………..

471) 8 months old infant with on & off recurrent crying episodes & hx of current
jelly stools:
a) intussception
b) intestinal obstruction
c) mickel's diverticulitis
d) strangulated hernia

The correct answer is a


472) a man with oblong swelling on top of scrotum increase in size with valsalva
maneuver most likely Dx:
a) direct inguinal hernia
b) indirect inguinal hernia
c) varicocele
d) femoral hernia

the correct answer is b

* Indirect hernia: Herniation of abdominal contents through the internal and then
external inguinal rings and eventually into the scrotum (in males).
- The most common hernia in both genders.
- Due to a congenital patent processus vaginalis.
- Indirect inguinal hernia increases in size it becomes apparent when the patient
coughs, and persists until reduced.

* Direct hernia: Herniation of abdominal contents through the floor of Hesselbach’s


Triangle. (Hesselbach’s triangle is an area bounded by the inguinal ligament, inferior
epigastricartery, and rectus abdominis)
- Hernial sac contents do not traverse the internal inguinal ring; they herniate
directly through the abdominal wall and are contained within the aponeurosis of
the external oblique muscle.
- Most often due to an acquired defect in the transversalis fascia from mechanical
breakdown that ↑ with age.

……………………………………………………………………………………………………………………………………………………..
473) 3 weeks old male newborn with swelling of scrotum transparent to light &
irreducible:
a) epidydemitis
b) hydrocele

The correct answer is b

……………………………………………………………………………………………………………………………………………………..
474) a young girl experienced crampy abdominal pain & proximal muscular
weakness but normal reflexes after receiving septra (trimethoprim
sulfamethoxazole) :
a) functional myositis
b) polymyositis
c) guillian barre syndrome
d) neuritis

the correct answer is d


475) Which drug causes SLE like syndrome:
a) hydralazine
b) propranolol
c) amoxicillin

The correct answer is a

- Drug-induced SLE causes: Chlorpromazine, Hydralazine, Isoniazid, Methyldopa,


Penicillamine, Procainamide, Quinidine, Sulfasalazine.

……………………………………………………………………………………………………………………………………………………..

476) When to give aspirin and clopidogrel?


a) pt with a hx of previous MI
b) Acute MI
c) hx of previous ischemic stroke
d) hx of peripheral artery disease
e) after cardiac capt

The correct answer is b

……………………………………………………………………………………………………………………………………………………..

477) 17 years with history of right iliac fossa pain rebound tenderness +ve guarding
what is the investigation that you will do?

a. Laparoscopy

b. US

c. CT scan

the correct answer is c

In appendicitis, CT scan with contrast has 95–98% sensitivity which shows


periappendiceal streaking.
478) Child with history of URTI before 3 weeks now has both knees tender, red and
inflamed. What is your diagnosis?

a. Rheumatoid arthritis

Q is not complete

The diagnosis of JRA is established by the presence of arthritis, the duration of the
disease for at least 6 weeks, and the exclusion of other possible diagnoses.
Although a presumptive diagnosis of systemic-onset JRA can be established for a
child during the systemic phase, a definitive diagnosis is not possible until arthritis
develops. Children must be younger than 16 years old at time of onset of disease;
the diagnosis of JRA does not change when the child becomes an adult. Because
there are so many other causes of arthritis, these disorders need to be excluded
before providing a definitive diagnosis of JRA (Table 89-2). The acute arthritides can
affect the same joints as JRA, but have a shorter time course. In particular, JRA can
be confused with the spondyloarthropathies, which are associated with spinal
involvement, and enthesitis, which is inflammation of tendinous insertions. All of
the pediatric spondyloarthropathies can present with peripheral arthritis before
other manifestations and initially may be diagnosed as JRA (Table 89-3).

* Juvenile rheumatoid arthritis: mono- and polyarthropathy with bony destruction


that occurs in patients ≤ 16 years of age and lasts > 6 weeks. Approximately 95% of
cases resolve by puberty. Subtypes: Pauciarticular: ≤ 5 joints involved, associated
with an  risk of iridocyclitis that lead to blindness if left untreated. Polyarticular:
≥ 5 small joints. Systemic features are less prominent; carries a  risk
of iridocyclitis. Acute febrile: The least common subtype; manifests as arthritis
withdaily high, spiking fevers and an evanescent, salmon-colored rash.
Hepatosplenomegaly and serositis may also be seen. No iridocyclitis is present;
remission occurs within one year.

……………………………………………………………………………………………………………………………………………………..
479) Old female came complain from pan in her joint increase with walking, what
is your diagnosis?
a. Osteoarthritis

Physical signs of OA: Crepitus,  range of motion, pain that worsens with activity
and weight bearing but improves with rest.
480) Patient came with history of neck discomfort, palpitation, cold sweat, TSH
low, T4 high, tender neck. What is your diagnosis?

a. Subacute thyroiditis.

- Thyroiditis: Inflammation of the thyroid gland. Common types are subacute


granulomatous, radiation, lymphocytic, postpartum, and drug-induced (e.g.,
amiodarone) thyroiditis.
- Hx/PE: In subacute and radiation, presents with tender thyroid, malaise, and URI
symptoms.
- Dx: Thyroid dysfunction (typically hyperthyroidism followed by hypothyroidism),
all with ↓ uptake on RAIU.
- Rx:
- β-blockers for hyperthyroidism; levothyroxine for hypothyroidism.
- Subacute thyroiditis: Anti-inflammatory medication.

……………………………………………………………………………………………………………………………………………………..
481) In ulcerative colitis, what you will start treatment with?
a. Corticosteroids ??

- Sulfasalazine or 5-ASA (mesalamine).


- Crticosteroids and immunosuppressants indicated if no refractory disease.

……………………………………………………………………………………………………………………………………………………..
482) Treatment of prostatitis?
a. Ciprofloxacin

treatment is for 4–6 weeks with drugs that penetrate into the prostate, such as
trimethoprim or ciprofloxacin.

……………………………………………………………………………………………………………………………………………………..
483) In IV canula and fluid:
a.Site of entry of cannula is a common site of infection.

……………………………………………………………………………………………………………………………………………………..
484) Patient with neck rigidity, rigor, fever, petechial rash over extremities. The
causative m\o is:
a. Meningococcal meningitis

Discuss meningitis !

……………………………………………………………………………………………………………………………………………………..
485) Female patient works in office came with vulva itching, yellow vaginal
discharge:
a. Trichomoniasis
486) another one old c/o bilateral knee pain with mild joint enlargement
ESR and CRP normal dx:
a. Osteoarthritis
b. Rheumatoid arthritis
c. Gout?

The correct answer is a

……………………………………………………………………………………………………………………………………………………..
487) .Scitica increased incidence of :
a. Lumbar lordosis
b. Parasthesis

Q not complete but with these MCQs , correct answer is b

……………………………………………………………………………………………………………………………………………………..
488) old male c/o knee pain on walking with crepitus xray show narrow joint space
and subchondoral sclerosis:
a. Rheumatoid arthritis
b. Osteoarthritis
c. Gout

The correct answer is b

……………………………………………………………………………………………………………………………………………………..

489) 14y f with BMI 32.6 (associated big chart):


a. Overweight
b. Obese
c. Normal weight

The correct answer is b

BMI < 16 : severe under WT.


BMI 16 – 20 : under wt.
BMI 20 – 25 : normal
BMI 25 – 30 : over wt.
BMI 30 – 35 : obese classic 1
BMI 35 – 40 : obese classic 2
BMI > 40 : obese classic 3
490) vaginal discharge odorless watry microscopy show clue cells :
a. Bacterial vaginosis
b. Candidiasis

The correct answer is a

……………………………………………………………………………………………………………………………………………………..

491) Pregnant women on 3rd trimester infected with measels can I give her
MMR?????

……………………………………………………………………………………………………………………………………………………..

492) lactating women infected with rupella ……..management is :


a. MMR
b. Stop lactation

The correct answer is a


MMR is safe during lactating

……………………………………………………………………………………………………………………………………………………..

493 ) which of following not live vaccine:


a. BCG
b. Hepatitis B
c. Oral polio
d. MMR

The correct answer is b

……………………………………………………………………………………………………………………………………………………..

493) the most commone cause of community acquired pneumonia:


a. Hemophilus influenza
b. Strept.pneumonia
c. Mycoplasma
d. Kliebsella

The correct answer is b


494) female after vaginal hesterectomy she complain of urin come from
vagina………dx:
a. Vesicovaginal fistula
b. Urethrovaginal fistula
c. Ureterovaginal fistula

The correct answer is a

……………………………………………………………………………………………………………………………………………………..

495) child rt ear pain and tenderness on pulling ear , no fever , O/E inflamed
odemateous rt ear canal with yellow discharge >>>>>>>>>>dx:
a. Otitis media
b. Otitis externa
c. Cholesteatoma

The correct answer is b

……………………………………………………………………………………………………………………………………………………..
496) male with perianal pain ,tenderness ,fluctuant (perianal abscess)
a. Incision and drainage
b. Warm bath
The correct answer is a

……………………………………………………………………………………………………………………………………………………..
497) . pregnant 30w with vaginal bleeding………. what would you ask about :
a. Cigarette smoking
b. Recent sexual intercourse
Q not complete , but with theses MCQs may be correct answer is a

……………………………………………………………………………………………………………………………………………………..
498) In duodenal obstruction of neoborn what is the sign that apper in xray:
a. Double bubble
499) the following more commone with type2 DM than type1 DM:
a. Weight loss
b. Gradual onset
c. Hereditary factors
d. HLA DR3+-DR4

The correct answer is c

……………………………………………………………………………………………………………………………………………………..
500) pt with open angle glaucoma and k/c of COPD and DM ttt:
a. Timelol
b. betaxolol
c. Acetazolamide

The correct answer is c

……………………………………………………………………………………………………………………………………………………..

501) pt child with back pain that wake pt from sleep So diagnosis
a. lumber kyphosis
b. osteoarthritis
c. RA
d. Scoliosis

The correct answer is a or d

…………………………………………………………………………………………………………………………………………

502) 59 y/o presented with new onset supraventicular tachycardia with


palpitation,no Hx of SO Or chest pain ,chest examination normal , oxygen sat in
room air = 98% no peripheral edemaOthers normal, the best initial investigation:

a. ECG stress test


b. Pulmonary arteriography
c. CT scan
d. Thyroid stimulating hormone

the correct answer is d


503) Lactating women presented with breast engorgment and tendress Your
managements:

a. Warm compressor and continoue breast feeding

b. Dicloxacillin and continoue breast feeding

c. Dicloxacillin and milk expression

d. Discontinoue breast feeding and cold compressor

the correct answer is a . tis case is ( engorgement of breast ) . but if there are redness
, swelling , fever this case is ( mastitis ) and choose b

504) The Fastest route of antipsychotic is:

a. IM

b. IV

c. Oral

d. Sublingual

the correct answer is b

505) While you are in the clinic you find that many patients presents with red follicular
conjactivitis (Chlamydia ) your management is:

a. Improve water supply and sanitation

b. Improve sanitation and destroying of the vector

c. Eradication of the reservoir and destroying the vector

d. Destroy the vector and improve the sanitation

the correct answer is a


506) The most important exogenous risk factor for osteoporosis is:

a. Alcohol intake

b. Age

c. Smoking

d. Lack of exercise

the correct answer is c

507) Patient with family history of coronary artery disease his BMI= 28 came to you
asking for the advice:

a. Start 800 calorie intake daily

b. Decrease carbohydrate daytime

c. Increase fat and decrease protein

d. Start with decrease ……. K calorie per kg per week

the correct answer is d

508) Lactating mother newly diagnosed with epilepsy , taking for it phenobarbital you
advice is:

a. Discontinue breastfeeding immediately

b. Breastfeed baby after 8 hours of the medication

c . Continoue breastfeeding as tolerated

the correct answer is c

-very vague question , some books avoid Phenobarbital during breast feeding if
possible. And in American academy of pediatric classified Phenobarbital as adrug
that cause major advers effect in some nursing infant, and should be given to
nursing women with cation .
509) Pregnant women has fibroid with of the following is True:

a. Presented with severe anemia


b. Likely to regress after Pregnancy
c. Surgery immediately
d. Presented with Antepartum He

the correct answer is b

Fibroids may also be the result of hormones. Reproductive hormones like


estrogen and progesterone can stimulate cell growth, causing fibroids to form.
During pregnancy, your influx of hormones may cause your fibroids to grow in
size. After pregnancy and during menopause most fibroids begin to shrink, due
to a lack of hormones.

510) A known case of chronic atrial fibrillation on the warfarin 5 mg came for follow
up you find INR 7 but no signs of bleeding you advice is:

a. Decrease dose to 2.5 mg


b. Stop the dose and repeat INR next day
c. Stop warfarin
d. Continoue same and repeat INR

the correct answer is b

INR ACTION

>10 Stop warfarin. Contact patient for examination. MONITOR INR

7-10 Stop warfarin for 2 days; decrease weekly dosage by 25% or by 1 mg/d for next week (7 mg
total); monitor INR

4.5-7 Decrease weekly dosage by 15% or by 1 mg/d for 5 days of next week (5 mg total); repeat
monitor INR

3-4.5 Decrease weekly dosage by 10% or by 1 mg/d for 3 days of next week (3 mg total); repeat
monitor INR.

2-3 No change.

1.5-2 Increase weekly dosage by 10% or by 1 mg/d for 3 days of next week (3 mg total);

<1.5 Increase weekly dose by 15% or by 1 mg/d for 5 days of next week (5 mg total);
511) Patient is a known case of CAD the best exercise:

a. Isotonic exercise

b. Isometric exercise

c. Anerobic exe

d. Yogha

the correct answer is c

1-anerobic exercise ( endurance ) : for improve cardiac function


2- weight bearing excercise ( isometric ) : for build muscle strength , bone density
3- stretching excercise : for prevent cramp , stiffness and back pain

512) The mechanism of action of Aspirin:

a. Inhibit cycloxgenase

b. Inhibit phospholipase A2

c. Inhibit phospholipid D

the correct answer is a

513) The absolute contraindication of breastfeeding is :

a. Asymptomatic HIV patient

b. Active hepatitis C

c. Pulmonary TB on treatment 3 months

The correct answer is a

Absolute contraindication of breastfeeding :

1-Infants with galactosemia.

2-Mothers who use illegal drugs.

3-Mothers infected with HIV, human T -cell lymphotropic virus type I


or type II, or who have an active herpes lesion on the breast.

4-Mothers taking any of the following medications: radioactive


isotopes, cancer chemotherapy agents, such as antimetabolites
.
514) A boy felt down on his elbow , the lateral x-ray shows:

a. Anterior Pad sign

b. Posterior pad sign

c. Anterior line of humerous intersecting the cubilium

d. Radial line forming 90 degree with cubilium

the correct answer is b

515) A known case of treated hodgkin lymphoma(mediastinal mass) with radiotherapy


Not on regular follow up presented with gradual painless difficulty in swallowing
and SOB , There is facial swelling and redness : DX

a. SVC obstruction

b. IVC obstruction

c. Thoracic aortic aneurysm

d. Abdominal aortic aneursm

The correct answer is a

________________________________________________________________________

516) Patient is presented with hand cellulitis and red streaks in the hand and tender
axillary lymphadenopathy. This condition is more likely to be associated with:

a. Malignancy

b. Pyoderma

c. Neuropathy

d. Lymphangitis
the correct answer is b (I'm not sure )
516) Young aged male presented to ER after blunt trauma to Abdomen, CT scan shows
intramural hematoma: your management is

a. Lapratomy with evacuation of the hematoma

b. Dissection of duodenum

c. Observation

the correct answer is c

517) Patient presented with sore throat, anorexia, loss of appetite , on throat exam
showed enlarged tonsils with petechi on palate and uvula , mild tenderness of
spleen and liver :DX

a. Group A strep
. b. EBV

The correct answer is b

518) Patient with GERD has barret esophagus , this metaplasia increase risk of :

a. Adenocarcinoma
b. Squmaou cell carcinoma

the correct answer is a

519) Complication of Sleep apnea is :

a.CHF
b. ……

the correct answer is a

sleep apnea : Hypoxic pulmonary vasoconstriction  PAH  Cor Pulmonale  CHF

complication of sleep apnea : sleep apnea increases health risks such as


cardiovascular disease, high blood pressure, stroke, diabetes, clinical depression, weight
gain and obesity. The most serious consequence of untreated obstructive sleep apnea is to
the heart. In severe and prolonged cases, there are increases in pulmonary pressures that
are transmitted to the right side of the heart. This can result in a severe form of congestive
heart failure (cor pulmonale).
520) Which of the following medication can be used as prophylaxis in appendectomy:

a. Cephalexin

b. Ceftriaxone

c. Metronidazole

d. Vancomycin

e. Ampicillin

The correct answer is b

1st line of antibiotic : 1- cefoxitin 2- cefotetan

2nd line of treatment : 1- metradinazole 2- ampicikkin-sulbactam

521) Which of the following prognostic factor for SLE:

a. ANA levels

b. Sex

c. Age

d. Renal involvement

The correct answer is d

522) The most common site for osteomyelitis is:

a. Epiphysis

b. Diaphysis

c. Metaphysis

d. Blood flow

The correct answer is c


523) In “holding breath holding” which of the following True:

a. Mostly occurs between age of 5 and 10


b. Increase Risk of epilepsy
c. A known precipitant cause of generalized convulsion
d. Diazepam may decrease the attack

Breath holding spells are the occurrence of episodic apnea in children, possibly associated with
loss of consciousness, and changes in postural tone. They are most common in children between
6 and 18 months and usually not present after 5 years of age. They are unusual before 6 months
of age. A positive family history can be elicited in 25% of cases. It may be confused with a
seizure disorder.

There are four types of breath holding spells.

1-The most common is termed simple breath holding spell, in which the manifestation is the
holding of breath in end expiration. There is no major alteration of circulation or oxygenation
and the recovery is spontaneous.

2-The second type are the Cyanotic breath-holding spells. They are usually precipitated by
anger or frustration although they may occur after a painful experience. The child cries and has
forced expiration sometimes leading to cyanosis (blue in color), loss of muscle tone, and loss of
consciousness. The majority of children will regain consciousness. The child usually recovers
within a minute or two, but some fall asleep for an hour or so. Physiologically, there is often
hypocapnea (low levels of carbon dioxide) and usually hypoxia (low levels of oxygen. There is no
"post ictal" phase (as is seen with seizures), no incontinence, and the child is fine in between
spells. EEGs are normal in these children. There is no relationship to the subsequent
development of seizures or cerebral injury as a conseque nce of breath holding spells.

3-In the third type, known as Pallid breath-holding spells, the most common stimulus is a
painful event. The child turns pale (as opposed to blue) and loses consciousness with little if any
crying. The EEG is also normal, and again there is no post ictal phase, nor incontinence. The child
is usually alert within a minute or so. There may be some relationship with adulthood syncope in
children with this type of spell.

4- A fourth type, known as Complicated breath-holding spells, may simply be a more severe
form of the two most common types. This type generally begins as either a cyanotic or pallid
spell that then is associated with seizure like activity. An EEG taken while the child is not having
a spell is still generally normal.

DX: clinical , good history include sequence of event , lack of incontinence and no post ictal
phase. Treatment : reassurance and iron.
524) Infant brought by the mother that noticed that the baby has decreasing feeding ,
activity and lethargic On examination febrile(39), tachycardic ,his bp 75/30, with
skin rash . DX:

a.Septic shock

Q not complete

525) Old patient newly diagnosed with hyperthyroidism presented with


(hyperthrodism symptoms) The best initial symptomatic treatment is:

a. BB ( beta-blocker )

b. PTU

The correct answer is a

_______________________________________________________________________

526) Infant presented with hemangioma on the back . your management is:

a. Intralesional injection of corticosteroids

b. Topical corticosteroids

c. Excise of the lesion

527) Pregnant lady , 34 wk GA , presented with vaginal bleeding more than her
menstruation. On examination , cervix is dilated 3 cm with bulging of the
membrane, fetal heart rate = 170 bpm . The fetus lies transverse with back facing
down . us done and shows that placenta is attached to posterior fundus and
sonotranulence behind placenta. Your management is :

a. C/S
b. Oxytocin
c.Tocolytics
d.Amniotomy
the correct answer is a
528) Infant with congenital hip dislocation:

a. +ve click in flexion ,abduction

b. The only treatment is surgery

c. Not reduced with flexion and abduction of the hip

the correct answer is a . this is ortolani test

529) In irritable bowel S. the following mechanism ?contraction and slow wave
myoelectricity seen in:

a.Constipation

b. Diarrhea

the correct answer is a

529) Which is not found in coarctation of the aorta:

a. Upper limb hypertension

b. Diastolyic murmur heard all over precordium

c. Skeletal deformity on chest x-ray

the correct answer is a (I'm not sure )

530) Female patient presented with tender red swelling in the axilla with history of
repeated black head and large pore skin in same area: ttt is

a. Immidate surgery

b. Topical antibiotic

c. Cold compressor

d. Oral antibiotic
531) In indirect hernia the relation of the sac to the cord structure is:

a. Anteromedial

b. Anterolateral

c. Posteromedial

d.Postrolateral

the correct answer is b

532) The most common cause of croup is:

a. Parainfluenza

b. Influenza

the correct answer is a

533) kwashikor disease usually associated with :

a. decrease protein intake, decrease carbohydrate

b. increase protein , increase carbo

c. decrease protein , increase carbo

the correct answer is c. in the book, written there is decrease protein and
adequate amount of carbohydrate, but with this only MCQs the answer is c

534) in cachectic patient, the body utilize the proteins of the muscles :

a. to provide Amino acid and protein synthesis

b. to maintain bloodflow to vital organ

c. to increase body fat

the correct anwser is a ( I'm not sure )

535) patient is complaining of memory loss. Alzehimer disease is diagnosed what is


the cause of this:

a. brain death cell

Q not complete
536) Parents brought their baby to you who is on bottle feeding. On exam whitish
lesion on either side of teeth seen with blackish lesion on maxillary incisors and
second molar teeth. There is history of leaving the baby with bottle in his mouth
during sleeping. The Dx:

a. Nursery dental caries

b.Gingvostomatis

the correct answer is a

537) Which of the following medication if taken need to take the patient immidiatly to
the hospital:

a. Penicillin

b. diphenhydramine

c. OCPs

d. Quinine or Quinidine

the correct answer is d (I'm not sure ) , quinidine is antiarrhythmia .

538) 43 y/o female presented with severe DUB other examination normal . your
management is

a. D &C

b. Ocps

c. Hysterectomy

d. Blood transfusion

the correct answer is a

539) Baby with vesicles on the face and honey comb crust which of the following
organism cause it: Staph aureus
540) Female patient presented with migraine headache which is pulsatile, unilateral ,
increase with activity . Dosn't want to take medication. Which of the following is
appropriate:

a. Bio feedback

b. TCA

c. BB

the correct answer is a

Biofeedback has been shown to help some people with migraines. Biofeedback is a
technique that can give people better control over body function indicators such as
blood pressure, heart rate, temperature, muscle tension, and brain waves. The two
most common types of biofeedback for migraines are thermal biofeedback and
electromyographic biofeedback.

541) Infant born with hemangioma on the rt eyelid what is appropriate time to
operate to prevent amylopia:

a. 1 day

b. 1 week

c. 3 months

d. 9 months

difficult Q. but may be the correct answer is b or c

542 ) Young patient on anti TB medication presented with vertigo which of the
following drug cause this:

a. Streptomycin

b. Ethambutol

c. Rifampcin
the correct answer is a , streptomycin cause 8th nerve damage.
543) The CPR for child is

a. 30 chest compression 2 ventilation … (Lone rescuer)

b. 15 chest c 2 ventilation … (2 or more rescures)

c. m15 chest compression 1 ventilation

the correct answer is a

544) Picture show large ulcer over medial side of the leg . what is your management

a. Shave biobsy

b. Elevate the legs and stocking

c. Topical steroids

the correct answer is b

……………………………………………………………………………………………………………………………………………………..
545) 2months infant with white plaque on tongue and greasy ,past h/o clamydia
conjunctivitis after birth treated by clinamycin what is ttt:
a. Oral nystatin
b. Topical steroids
c. Topical acyclovair
d. Oral tetracycline
the correct answer is a. oral nystatin : antifungal

……………………………………………………………………………………………………………………………………………………..
546) child rt ear pain and tenderness on pulling ear , no fever , O/E inflamed
odemateous rt ear canal with yellow discharge >>>>>>>>>>dx:
a. Otitis media
b. Otitis externa
c. Cholesteatoma

the correct answer is a

……………………………………………………………………………………………………………………………………………………..
547) 34y female with HIV pap smear negative, about cervical cancer screening :

a. After 3m if negative repeat after 6m


b. After 6m …………………………… annually
c. After 1y………………………………….annually

The correct answer is b ( I'm not sure , confuse with choice c )


548) female about 30y with breast cancer (given cbc –chem. And reavel low hb and
hematocrite….) what is the next step in mangment:
a. Staging
b. Lumpectomy
c. Mastectomy
d. chemotherapy

the correct answer is a

……………………………………………………………………………………………………………………………………………………..
549) child with moderate persistant BA On bronch.dilat inhaler. Presented with acute
exacerbation what will you add in ttt:
a. Corticosteroid inhaler
b. Ipratropum bromide inhaler

the correct answer is b

……………………………………………………………………………………………………………………………………………………..

550) about head and neck injury :


a. Hoarsness of voice and stridor can occure with mid facial injury
b. Tracheostomies contraindicated
c. Facial injury may cause upper air way injures

……………………………………………………………………………………………………………………………………………………..
551) lactating women 10 days after delivary developed fever ,malaise, chills tender Lt
breast with hotness and small nodule in upper outer quadrant with axillary LN
.Leucocytic count was 14 *10/L dx:
a. Inflammatory breast cancer
b. Breast abscess
c. Fibrocystic disease

the correct answer is b


552) 70y male with osteoporosis the T score of bone densometry would be :
a. -3.5
b. -2.5
c. 1
d. 2
e. 3.5

the correct answer is b

……………………………………………………………………………………………………………………………………………………..

553) child with epistaxis…….. management:


a. Compression on nose and leaning forward
b. ………………………………………………….backward

the correct answer is a

……………………………………………………………………………………………………………………………………………………..
554) scenario about female underwent abdominal operation she went to physician
For check……….. U/S reveal metal thing inside abdomen (a.e missed during operation)
What will you do :
a. Call the surgeon and ask him what to do
b. Call attorney and ask about legal action
c. Tell her what you found
d. Tell her that is one of possible complications of operation
e. Don't tell her what you found

The correct answer is c

……………………………………………………………………………………………………………………………………………………..
555) male ptn with scaly fine papular rash on fornt of scalp,nose and
retroauricular……..(i think tinea capitis) ttt is:
a. Ketoconazole cream…
b. Oral augmentin
c. ……… cream

the correct answer is a


556) All can cause gastric ulcer except:
a- Tricyclic antidepressant.
b- Delay gastric emptying.
c- Sepsis.
d- Salicylates.
e- Gastric outlet incompetent.

The correct answer is a


Tricyclic antidepressant use in treatment of peptic ulcer.
……………………………………………………………………………………………………………………………………………………..

557) 48year old female lost her menstruation for 2 cycles, the method of
contraception is condom, examination was normal except for dusky
discoloration of the cervix. What u will do next:
a.Progesterone challenge.
b.Beta HCG.
c.Pelvic u/s

The correct answer is b

……………………………………………………………………………………………………………………………………………………..

558) ttt of yeast vaginitis: Clotrimazole , Fluconazole , Ketokonazole

Treatment of yeast vaginitis ( candida albicans ) : topical azole or PO fluconazole ,


oral azole should be avoided in pregnant.

……………………………………………………………………………………………………………………………………………………..

559) 15 y/o female complaining of pain during menstruation, not sexually active,
medical hx unremarkable, physical examination normal, how to treat:
a- NSAID.
b- Danazole.
c-

The correct answer is a

……………………………………………………………………………………………………………………………………………………..

560) What is true about alpha blocker:


a. Causes hypertension.
b. Worsen benign prostatic hyperplasia.
c. Cause tachycardia.
d-

the correct answer is c


alpha blocker :cause orthostatic hypotension and tachycardia.
561) cyclic menstruation that increase in frequency:
a. Polymenorrhea.
b. Hypermenorrhea.
c. Menorrhagia.
d. Dysmenorrhea.
e.

The correct answer is a


Polymenorrhea : frequent menestration ( < 21 day cycle )
Hypermenorrhea or menorrhagia : abnormally heavy and prolonged menstrual
period at regular intervals . more than 80 ml of blood loss per cycle or prolonged
bleeding , more than 8 days.
Oligomenorrhea : increase length of time between menses ( 35-90 days b\w cycle)
Metrorrhagia : bleeding bwteen period.
Menometrorrhagia : exessive and irreglar bleeding.

……………………………………………………………………………………………………………………………………………………..

562) A pregnant lady came to you to screen her fetus for down syndrome, what is the
best method:
a. Amniocentesis. + Karyotyping
b. Choriocentesis.
c.

the correct answer is a

……………………………………………………………………………………………………………………………………………………..

563) What is the most common chromosomal abnormality?


a. Trisomy 13
b. Trisomy 21

the correct answer is b


-down syndrome ( trisomy 21 ) is the most common chromosomal abnormality.

……………………………………………………………………………………………………………………………………………………..

564) Best 2 test to screen for hepatocellular carcinoma:


a. Liver biopsy and alpha-fetoprotein.
b. Liver ultrasound and alpha-fetoprotein.
c. Abdomen CT and
d.

The correct answer is b


565) A child came to ER with fever, stridor, … , x-ray showed swollen epiglottis, in
addition to oxygen, what u will do?
a. Throat examination.
b. An emergency tracheostomy.
c. Endotracheal intubation.
d. Nasopharyngeal intubation.

The correct answer is c


……………………………………………………………………………………………………………………………………………………..

566) 24 y/o female newly diagnosed type 2 DM, she is wearing glasses for 10 years,
how frequent she should follow with ophthalmologist:
a. Every 5 years.
b. Annually

The correct answer is b


-for type 1 diabetic : retina screening annually beginning 5 years after onset of
diabetes, general not before onset of puberty.
- for type 2 diabetic : screening at the time of diagnosis then annual
……………………………………………………………………………………………………………………………………………………..

567) What is the initial management for a patient newly diagnosed knee
osteoarthritis.
a. Intra-articular corticosteroid.
b. Reduce weight.
c. Exercise.
d. Strengthening of quadriceps muscle.

The correct answer is b


……………………………………………………………………………………………………………………………………………………..

568) A lady came to your clinic said that she doesn’t want to do mammogram and
preferred to do breast self- examination, what is your response?
a- Mammogram will detect deep tumor.
b- Self-examination and mammogram are complementary.
c- Self-examination is best to detect early tumor

answer is b ( I'm not sure because MCQs not complete and depend on age of pt.

……………………………………………………………………………………………………………………………………………………..

569) What is the best frequency for breast self-examination?


a. Daily.
b. Weakly.
c. Monthly.
d. Annually.

The correct answer is c


570) Patient with left bundle branch block will go for dental procedure , regarding
endocarditis prophylaxis:
a. No need
b. Before procedure.
c. After the procedure.
d.

The correct answer is a


……………………………………………………………………………………………………………………………………………………..

571) classical characteristic for genital herpes.


Painful ulcers & vesicles

……………………………………………………………………………………………………………………………………………………..

572) most common vaginal bleeding :


a. cervical polyps
b. menstruation
c. ????

only with these MCQs , the correct answer is b


……………………………………………………………………………………………………………………………………………………..

573) best stimulant for breast milk secretion:

a. breast feeding
b. oxytocin
c. ???

the correct answer is a


……………………………………………………………………………………………………………………………………………………..

574) child with aspirin intake overdose ...what kind of acid base balance:
a. metabolic alkalosis wt respiratory
b. metabolic acidosis wt respiratory alkalosis
c. respiratory alkalosis with metabolic acidosid
d. respiratory acidosis with metabolic alkalosis

The correct answer is c


Aspirin toxicity : in early stages, salicylate will stimulate respiratory center 
RR  respiratory alkalosis that will be compensated by metabolic acidosis. In
late stage, it will interfere with COH, fat, & protein metabolism as well as
Oxidation phosphorylation lead to  lactate, pyrovate, & keton bodies. All will lead
to pH. Signe & symptoms includes: nausea, vomiting, RR, temp, HR,
sweating, cerebral or pulmonary edema, & coma. +ve anion gap. TTT: hydration,
correct K+, gastric lavage or activated charcoal, urine alkalization, hemodialysis)
575) therapeutic range of INR :
a. 2.5-3.5
b. 2.0-3.0
c. ???

The correct answer is b

……………………………………………………………………………………………………………………………………………………..

576) q about antidepressant:


a. start single type even patient have sever depression
b. start any one of them they all have the same efficacy
c. stop the medication after 2 weeks if no improvement

The correct answer is a

……………………………………………………………………………………………………………………………………………………..

577) patient with depression started on amitryptaline , he had headache or dizzness ,


vomitting?? im not sure what exactly was the symptoms
a. change to SSRI
b. ??

Q not complete , but amitriptyline is TCA .

……………………………………………………………………………………………………………………………………………………..

578) patient had horsenss of voice for 3 weeks... next to do:


a. throat swab
b. laryngoscopy
c. ???

The correct answer is b , I'm not sure because MCQs are not complete.

……………………………………………………………………………………………………………………………………………………..

579) single diagnostic for stroke:


a. high cholesterol
b. high systolic blood pressur

the correct answer is b , I'm not sure , but with these mcqs , b is correct.
580) unfaivrable prognosis for schezophrenia:

a. family Hx

b. failed marrige

c. adolscen age?? not sure

d. presence of psychosis

the correct answer is a


……………………………………………………………………………………………………………………………………………………..

581) Rt lung :
a. have 1 fissure
b. contain 7 segment
c. ??? read about lung anatomy segment names!!

Rt lung has 3 lobes & 10 segments (medial basal segment only in inf. Lobe of Rt lung)
Lft lung has 2 lobes & 9 segments (Lingular segments in lft lung)

……………………………………………………………………………………………………………………………………………………..

582) q about specifity:


a. test is +ve in disese population
b. test is +ve in healthy poplation
c. test is -ve in diseased population
d. test is -ve in healthy population

The correct answer is d


……………………………………………………………………………………………………………………………………………………..

583) stage 3 colon ca  Give Chemo ASAP

……………………………………………………………………………………………………………………………………………………..

584) patient with upper abdominal pain, nausea vomitting,with back pain, he is
smoker for long time daily, fecal fat was +ve
a. acute pancreatitis
b. chronic pancreatitis
c. pancreatic CA

The correct answer is b


585) in school, 10 of patient had the dis of the first week , then 30 pt. had the dieses
the next week , the percentage infection of this school is:
a. 10%
b. 20%
c. 30%
d. 40%

Q is not clear or not complete, but I think the correct answer is b.


……………………………………………………………………………………………………………………………………………………..

586) patient had headche describe it as aband around his head, increase with stress
and , i dont remeber the ques DX is:
a. tension headache
b. migraine
c. cluster headache

The correct answer is b

……………………………………………………………………………………………………………………………………………………..
587) Patient was presented by bollus in his foot , biopsy showed sub dermal lysis ,
fluorescent stain showed IgG , what is the most likely diagnosis :
A. Bolus epidermolysis .
B. Pemphigoid vulgaris .
C. Herpetic multiform .
D. Bullous pemphigoid .

The correct answer is d

……………………………………………………………………………………………………………………………………………………..

588) Patient was presented by difficulties of breathing from one side of his nose , on
examination there was erythramatus swelling , what is the best initial treatment :
A. Decongestant .
B. Steroid .
C. Sympathomimetics . ??

The correct answer is c ( I'm not sure )

Explain: this swelling might be nasopharyngeal angiofibroma which is highly vascular


swelling & prone to epistaxis, so to decrease the size we might use local or systemic
decongestants which are sympathomimetics, so answer is not clear.
589) Patient with cystic nodule (acne) and scars , what is the best treatment :
A. Retinoin .
B. Erythromycin .
C. Doxycyclin .

The correct answer is a ( I'm not sure )

this seems to be an advanced inflammatory stage of acne since there r scars so if


the retinoiD meant here is systemic (isotretinin) so it will be the choice. If topical it
will not be effective as it is effective only in the non-inflammatory stage
(comedones). Both erythromycin (local) & doxycyclin (systemic) can be used but in
advanced stage systemic is better.

……………………………………………………………………………………………………………………………………………………..

590) Patient was presented by constipation, vomiting , abdominal distension , with


old scar in the lower abdomen , x ray showed dilated loops with air in the rectum
, what is the best initial management :
A. NGT decompression , and IV line .
B. Rectal decompression and antibiotics .
C. Suppositories .

The correct answer is a

this is a case of intestinal obstruction due to adhesions from previous abdominal


surgery (commonest cause) so the initial treatment would be decompression of the
intestine & replace lost fluids

……………………………………………………………………………………………………………………………………………………..

591) Which of the following is not living vaccine :


A. Hepatitis B .
B. MMR .
C. Oral polio
D. BCG

The correct answer is a

Hepatitis B is the only vaccine prepared by genetic engineering so its not living vaccine.
592) Six years old child was born to a mother with hepatitis B , he does not received
any vaccines before , what you will give now :
A. DTP , MMR , Hib
B. DTP , MMR .
C. DT , MMR , Hib
D. DT, MMR .

The correct answer is d

both Pertussis & H.influenza vaccine are not needed after 1 year of age.

……………………………………………………………………………………………………………………………………………………..

593) Which of the following describes the end of the early inflammatory phase .
A. Formation of eschar .
B. Formation of ground base of collagen .
C. The end of angiogenesis .

The correct answer is c

……………………………………………………………………………………………………………………………………………………..

594) Female patient , known case of VitD deficiency , smoking , and recurrent fall,
which of the following is the greatest exogenous risk for osteoporosis :
A. Advanced age .
B. Recurrent fall .
C. Vit D .
D. Smoking .

The correct answer is d

Advanced age & female gender are the most important factors for osteoporosis but
they are endogenous. Smoking is an independent exogenous risk factor for
osteoporosis
595) Blood sugar in DM type 1 is best controlled by :
A. Short acting insulin .
B. Long acting .
C. Intermediate .
D. Hypoglycemic agents .
E. Basal and bolus insulin .

The correct answer is e

Very vague question. We can exclude hypoglycemic agents. Short acting insulin is
best in emergencies like DKA as it can be given IV. We can use either long acting
alone daily or a mixture of short & intermediate acting insulin daily. Basal & bolus ,
( short acting + intermediate or long ), bolus’ of short-acting or very-short-acting
insulin before meals to deal with the associated rise in blood-sugar levels at these
times. In addition, they take an evening injection of long- or intermediate-acting
insulin that helps normalise their basal (fasting) glucose levels. This offers greater
flexibility and is the most commonly adopted method when intensified insulin
therapy is used to provide optimal glycaemic control.

……………………………………………………………………………………………………………………………………………………..

596) Which of the following is true regarding perths disease :


A. Commonly seen between 11-16 years of age .
B. Always unilateral .
C. May present by painless limp .
D. Characteristically affect the external rotation of hip .
E. More in female .

The correct answer is c

Perthes disease (Leg-Calve perthes disease) is a condition affecting the hip joint
where there is degenerative avascular necrosis of the femoral head. It affects
children aged 3-12 years & is more common in males. It is most commonly but not
always unilateral ( 85% is unilateral ). It presents mainly by severe hip pain &
limping that increases by movement but it can present by painless limp. It
characteristically affects the internal rotation & abduction of the hip & limits these
movements.
597) Which of the following is true regarding gastric lavage :
A. Patient should be in the right lateral position .
B. It is not effective after 8 hours of aspirin ingestion .

The correct answer is b

lavage is effective only 1 hour after ingestion of any poison. After that its ineffective

……………………………………………………………………………………………………………………………………………………..

598) SSRI was prescribed to a patient with depression , the effect is suspected to be
within :
A. One day .
B. Two weeks .
C. Three to four weeks .

The correct answer is c


Allow 2 – 6 weeks to take effect , and treat for > 6 months

……………………………………………………………………………………………………………………………………………………..

599) Randomized controlled study will be stronger by :


A. Systemic >>>>
B. Following at least 50 % of the participant .

???????

……………………………………………………………………………………………………………………………………………………..

600) Partner lost his wife by AMI 6 months ago , presented by loss of appetite , low
mood , sense of guilt , what is the diagnosis :
A. Beverament .
B. Major depression episode .

The correct answer is a

- Major depression is a psychiatric condition that occurs regardless of events that


happen in life, while normally most people would have beverament after death of a
close person.

You might also like